Psychiatry Test Preparation and Review Manual, 7 edition

Chapter 3: Test Number Three

1. During which one of Piaget’s stages of development will a child be able to understand that a tall glass and a short wide glass can contain the same volume of water despite their different shapes?

A Sensorimotor stage

B Preoperational thought stage

C Concrete operations stage

D Formal thought stage

E Anal stage

2. Which one of the following is not a characteristic of Transient global amnesia?

A Loss of personal information and identity

B Reversible anterograde and retrograde memory loss

C Inability to learn newly acquired information

D Preservation of alertness without motor or sensory deficits

E Men are affected more commonly than women

3. Which one of the following answer choices is not true regarding GABA?

A GABA is thought to suppress seizure activity

B GABA is thought to exacerbate mania

C GABA is thought to decrease anxiety

D GABA-A activity is potentiated by topiramate

E Gabapentin has no activity at the GABA receptors or transporter

4. An 8-year-old boy has been at sleep-away summer camp for 2 weeks and presents with a sudden onset of facial diplegia. The most likely infectious organism that might have caused this symptom is:

A Treponema pallidum

B Borrelia burgdorferi

C Leptospira interrogans

D Rickettsia rickettsii

E Yersinia pestis

5. A middle-aged man has been referred to your office by a plastic surgeon. The man is seeking a face lift for his “excessively large cheeks”. The surgeon has not been able to find anything abnormal about the man’s face or skin, and when he comes to see you, you fail to see anything wrong either. The patient insists that his cheeks are grotesque and ruining his whole appearance. When pressed, he admits that others may not consider his cheeks to be as bad as he does. His most likely diagnosis is:

A Malingering

B Schizophrenia

C Somatization disorder

D Conversion disorder

E Body dysmorphic disorder

6. Tourette’s syndrome often involves which one of the following psychiatric manifestations?

A Generalized anxiety disorder

B Social anxiety disorder

C Panic disorder

D Obsessive–compulsive disorder

E Psychotic disorder

7. A young woman comes to the emergency room with a 1 week history of pressured speech, decreased sleep, grandiosity, and loosening of associations. The patient feels that she is being monitored by a satellite and she is seen talking to herself when no one else is in the room. Which one of the following criteria must be met to diagnose this patient with schizoaffective disorder instead of bipolar disorder?

A Presence of mania

B Psychotic symptoms in the absence of mood symptoms for a 1 week period

C At least one prior episode of depression

D Presence of psychotic symptoms during a manic episode

E Psychotic symptoms in the absence of mood symptoms for a 2 week period

8. Which one of the following therapies would be best-suited to a bipolar patient in a manic episode during pregnancy?

A Haloperidol

B Lithium

C Aripiprazole

D Divalproex sodium

E Electroconvulsive shock therapy

9. You are called to evaluate a 60-year-old man with a history of depression. His family reports that he has not been himself for the past 5 days. On examination he makes poor eye contact, is inattentive, mutters incoherently, keeps rearranging pieces of paper on his bed tray with no apparent logic, and drifts off to sleep while you are talking to him. What is his most likely diagnosis?

A Depression

B Dementia

C Delusional disorder

D Delirium

E Obsessive–compulsive disorder

10. Which one of the following pharmacologic agents would be most likely to cause extrapyramidal side effects and possible tardive dyskinesia?

A Hydroxyzine

B Diphenhydramine

C Metoclopramide

D Ondansetron

E Tizanidine

11. The most common symptom seen in patients with narcolepsy is:

A Hypnopompic hallucinations

B Sleep paralysis

C Hypnagogic hallucinations

D Sleep attacks

E Cataplexy

12. The primary auditory cortex localizes to which one of the following brain regions?

A Temporal lobe

B Parietal lobe

C Frontal lobe

D Occipital lobe

E Thalamus

13. Which one of the following neurotransmitters works as an adjunctive neurotransmitter for glutamate as well as an independent neurotransmitter with its own receptors?

A GABA

B Norepinephrine

C Serotonin

D Dopamine

E Glycine

14. Which one of the following agents is not acceptable and useful for the treatment of postherpetic neuralgia?

A Gabapentin

B Lidoderm

C Pregabalin

D Carbamazepine

E Phenytoin

15. A construction worker is brought to the emergency room immediately after an accident on a job site. He was standing very near a three story scaffold that fell and missed crushing him by inches. He reports feeling anxiety, a sense of numbing, detachment, difficulty remembering the accident, and states that he feels like he is in a daze. The most likely diagnosis is:

A Generalized anxiety disorder

B Major depression

C Delirium

D Dissociative amnesia

E Acute stress disorder

16. Trauma to which one of the following vessels or groups of vessels commonly causes epidural hematoma?

A Middle meningeal artery

B Meningeal bridging veins

C Cavernous sinus

D Basilar artery

E Transverse sinus

17. A 26-year-old woman comes into the emergency room. She reports that she has been having mood swings that go from depressed to elated to rageful in minutes to hours. She has been having paranoid feelings and vague auditory hallucinations over the past week since breaking up with her boyfriend. On this past Monday she cut her arms with a razor, but only superficially. Her history reveals promiscuity, unstable relationships, and cocaine use. She now reports suicidal ideation. Her most likely diagnosis is:

A Bipolar disorder

B Depression with psychotic features

C Schizoid personality disorder

D Borderline personality disorder

E Schizotypal personality disorder

18. What is the characteristic electroencephalographic pattern noted in Creutzfeldt–Jakob disease?

A Three-per-second spike and wave

B Periodic high-amplitude sharp wave complexes

C Temporal spikes

D Generalized background slowing

E Periodic lateralizing epileptiform discharges (PLEDS)

19. To be diagnosed with polysubstance dependence, how many substances must a patient be found dependent on?

A Six substances, including caffeine

B Two groups of substances, where dependence criteria were met for one of the groups

C Four substances, including nicotine, but excluding caffeine

D Five specific substances

E Three or more groups of substances where dependence criteria were met for the groups, but not necessarily for any one particular substance

20. Which one of the following cerebrospinal fluid findings is indicative of aseptic meningitis?

A Variably increased lymphocytes, slightly decreased glucose, very high protein

B Moderately increased lymphocytes, decreased glucose, mildly elevated protein

C Highly increased neutrophils, decreased glucose, very high protein

D Slightly increased lymphocytes, normal glucose, mildly elevated protein

E Absent or few lymphocytes, normal glucose, very high protein

21. A small child is in the park with her mother. As the two interact, the child goes off to play for a brief time, then returns to her mother, then goes off to play, then returns to her mother. The child continues this pattern, regularly checking to see that the mother is still there. She would best fit into which one of Mahler’s stages of separation–individuation?

A Normal autism

B Symbiosis

C Rapprochement

D Practicing

E Object constancy

22. What type of systemic poisoning results in the development of characteristic Mees’ lines of the fingernails as in the photograph?

A Mercury

B Arsenic

C Lead

D Organophosphates

E Ionizing radiation

image

23. The assumption that there is no significant difference between two random samples of a population is called:

A Correlation coefficient

B Control group

C Analysis of variance (ANOVA)

D Regression analysis

E Null hypothesis

24. Organophosphate poisoning by pesticide exposure causes neurologic deficits by which one of the following mechanisms?

A Anticholinergic toxicity

B Cholinergic toxicity

C Gamma-aminobutyric acid blockade

D Serotonergic toxicity

E Dopaminergic toxicity

25. Emil Kraepelin used which one of the following terms in reference to schizophrenia?

A Language delay

B Dementia praecox

C Split-personality

D Rebound hyperactivity

E Downward drift

26. Which one of the following agents causes poisoning in humans that can result in a blue line at the gingival margin?

A Manganese

B Thallium

C Arsenic

D Mercury

E Lead

27. The police bring a man into the hospital who has been stealing satellite dishes off of houses and setting them up in his own yard because he feels that he has a chip in his head that allows him to talk directly to God. He states that God has instructed him to do this as preparation for the second coming. When his wife is questioned about her husband’s behavior she responds that indeed God has been directly communicating with her husband, and that she has helped him steal some of the larger satellite dishes. The wife’s condition can best be described as:

A Schizoid personality disorder

B Delusional disorder

C Shared psychotic disorder

D Bipolar disorder

E Substance-induced psychotic disorder

28. Truncal ataxia or instability can result most specifically from a lesion to the:

A Cerebellar hemispheres

B Cerebellar vermis

C Cerebellopontine angle

D Thalamus

E Midbrain

29. Which one of the following neurotransmitters has large numbers of receptors in the spinal cord, is synthesized primarily from serine, and has been the subject of research involving negative symptom reduction in schizophrenia?

A GABA

B Glycine

C Serotonin

D Dopamine

E Glutamate

30. Which one of the following is not a contraindication to lumbar puncture?

A Thrombocytopenia

B Cerebral mass lesion

C Suspected meningitis with obtundation

D Recent head trauma

E Papilledema

31. What is the difference between post-traumatic stress disorder and acute stress disorder?

A The nature of the trauma

B The symptoms that follow the trauma

C The impairment resulting from the symptoms

D The duration of the symptoms

E The age of the patient

32. Which chromosomal abnormality is the most common cause of mental retardation?

A Trisomy 21

B Trisomy 18

C Cri-du-chat syndrome

D Fragile X syndrome

E Prader–Willi syndrome

33. Ending up in strange places with no recollection of how one got there, or finding objects in one’s possession that one doesn’t recall acquiring is most characteristic of:

A Anxiety

B Psychosis

C Histrionic personality

D Dissociation

E Depression

34. Which one of the following child abuse injuries is the most likely to result in death or long-term sequelae?

A Embolic stroke from multiple bone fractures

B Subdural hematoma from head trauma

C Skull fracture from head trauma

D Cerebral hypoxia from choking

E Seizures from head and brain trauma

35. A 59-year-old man comes into your office complaining of depression. His wife of 25 years died unexpectedly 5 weeks ago. Since then he has been crying, has had little appetite but has lost no weight, and reports difficulty sleeping. He has been going out to dinner once each week with friends and says that it helps get his mind off of his wife’s death. He is not suicidal. This is the first time in his life that he has had symptoms such as these. His most likely diagnosis is:

A Bipolar disorder

B Major depressive disorder

C Acute stress disorder

D Bereavement

E Dysthymic disorder

36. Which one of the following is not considered a lower motor neuron sign?

A Hypotonia

B Muscle atrophy

C Fasciculations

D Babinski’s sign

E Hyporeflexia

37. Mahler’s stage that is characterized by a baby considering himself a fused entity with his mother, but developing increased ability to differentiate between the inner and outer world is called:

A Normal autism

B Symbiosis

C Differentiation

D Rapprochement

E Object constancy

38. Which one of the following therapeutics can eliminate benign paroxysmal positional vertigo?

A Diazepam

B Brandt–Daroff exercises

C Meclizine

D Metoclopramide

E Gabapentin

39. Which one of the following is not a diagnostic criterion for kleptomania?

A Recurrent failure to resist stealing objects

B Decreased sense of tension immediately preceding the theft

C Pleasure at the time of committing the theft

D The theft is not done to express anger

E The act is not in the context of an antisocial personality disorder

40. The persistent vegetative state is not characterized by which one of the following?

A Preserved eye opening

B Preserved response to noxious stimuli

C Preserved eye tracking

D Preserved swallowing

E Preserved sleep–wake cycles

41. An Hispanic man comes into the emergency room complaining of headache, insomnia, fear, anger, and despair. Your differential diagnosis is most likely to include which one of the following?

A Schizophrenia

B Schizoaffective disorder

C Panic disorder

D Ataque de nervios

E Myoclonic sleep disorder

42. What is the most typical effect of depression on nocturnal sleep?

A Decreased total sleep time

B Initial insomnia

C Middle insomnia

D Early morning awakening

E Sleep–wake cycle reversal

43. Which one of the following symptoms would not be expected in a patient with pyromania?

A Deliberate and purposeful fire setting

B Tension before the act

C Fascination with fire

D Pleasure when setting fires

E Setting fires for monetary gain or as an expression of political ideology

44. Apoptosis mediated by the N-methyl-D-aspartate (NMDA) receptor complex is most likely caused by elevated intracellular levels of which one of the following ions?

A Calcium

B Magnesium

C Sodium

D Potassium

E Chloride

45. A patient is brought into the emergency room following a fight with police. Upon examination the psychiatrist finds that the patient has a history of several discrete assaultive acts. His aggression in these situations was out of proportion to what one would consider normal. The patient has no other psychiatric disorder and no history of substance abuse. He has no significant medical history. What is his most likely diagnosis?

A Impulse control disorder NOS

B Pyromania

C Mania

D Temporal lobe epilepsy

E Intermittent explosive disorder

46. A 45-year-old woman comes to the emergency room by ambulance unconscious and barely breathing. Paramedics found an empty bottle of 90 tablets of 2 mg clonazepam on her dresser that was filled at the pharmacy the day before. One of the first agents to administer to this patient in the acute setting is:

A Naloxone

B Flumazenil

C Dimercaprol

D Atropine

E Epinephrine

47. Which one of the following should not be considered in the differential diagnosis for intermittent explosive disorder?

A Delirium

B Dementia

C Temporal lobe epilepsy

D Obsessive–compulsive disorder

E Substance intoxication

48. The electroencephalographic pattern that most often characterizes infantile spasms is:

A Three-per-second spike and wave

B Hypsarrhythmia

C Periodic lateralizing epileptiform discharges (PLEDS)

D Triphasic sharp waves

E Burst-suppression pattern

49. Which one of the following psychiatric symptoms is not found with AIDS?

A Progressive dementia

B Personality changes

C Heat intolerance

D Depression

E Loss of libido

50. Which one of the following structures is not part of Papez’ circuit?

A Amygdala

B Mamillary body

C Fornix

D Cingulate gyrus

E Hippocampus

51. A child is in the doctor’s office for an evaluation. His mother is waiting outside in the waiting area. The child is aware that his mother still exists even though she is not present in the room. For this to be true, the child must have reached which one of Mahler’s stages of separation–individuation?

A Normal autism

B Practicing

C Differentiation

D Symbiosis

E Object constancy

52. Which one of the following neurologic disorders is not believed to be caused by defects in the calcium channel system?

A Lambert–Eaton myasthenic syndrome

B Malignant hyperthermia

C Hypokalemic periodic paralysis

D Familial hemiplegic migraine

E Benign familial neonatal convulsions

53. A young woman comes to her psychiatrist’s office for help. She feels that others are exploiting her, but has no hard evidence. She is preoccupied with the lack of loyalty that she feels all of her friends have. She reads hidden demeaning connotations into the psychiatrist’s comments. She bears grudges and is unforgiving of slights. Her most likely diagnosis is:

A Schizophrenia

B Schizotypal personality disorder

C Paranoid personality disorder

D Schizoid personality disorder

E Dementia

54. Which one of the following is not a clinical feature of phenylketonuria?

A Sensorineural deafness

B Infantile spasms

C Microcephaly

D A characteristic “mousy” odor

E Light hair and skin pigmentation

55. Which one of the following dopaminergic tracts or areas is responsible for the parkinsonian side effects of antipsychotic medications?

A Mesolimbic–mesocortical tract

B Tuberoinfundibular tract

C Nigrostriatal tract

D Caudate neurons

E Ventral striatum

56. Which one of the following disorders is correctly depicted in the photomicrograph below?

A Lissencephaly

B Schizencephaly

C Dandy–Walker syndrome

D Arnold–Chiari type I malformation

E Arnold–Chiari type II malformation

image

57. A schizophrenic man on an inpatient psychiatric unit develops a fever of 102.3°F (39.1°C), a high WBC count, unstable pulse and blood pressure, and rigidity in the arms and legs. The most likely diagnosis is:

A Meningitis

B Serotonin-specific reuptake inhibitor withdrawal

C Lithium toxicity

D Neuroleptic malignant syndrome

E PCP use

58. Internal carotid artery stenosis should be treated surgically by endarterectomy if the stenosis is symptomatic and above:

A 50%

B 60%

C 70%

D 80%

E 90%

59. A patient comes into the clinic because his family (whom he only sees one or two times per year) keeps telling him to “go see a shrink”. He has no close relationships. He participates mainly in solitary activities. He has no desire for sexual activity with others. He is indifferent to the praise or criticism of others. On examination his affect is flat. His most likely diagnosis is:

A Schizoid personality disorder

B Schizotypal personality disorder

C Narcissistic personality disorder

D Major depressive disorder

E Dysthymic disorder

60. Poor outcome in Guillain-Barré syndrome is often associated with a preceding infection by which one of the following pathogens?

A Haemophilus influenzae

B Streptococcus pneumoniae

C Eescherichia coli

D Campylobacter jejuni

E Clostridium difficile

61. Which one of the following is false regarding the Pearson correlation coefficient?

A It spans from −1 to +1

B A positive value means that one variable moves the other variable in the same direction

C It can give information about cause and effect

D It indicates the degree of relationship

E A negative value means that one variable moves the other variable in the opposite direction

62. Which one of the following solid tumors metastasizes most frequently to the brain?

A Breast

B Colon

C Prostate

D Lung

E Thyroid

63. A pervasive pattern of social inhibition, feelings of inadequacy, and hypersensitivity to negative evaluation is most characteristic of:

A Obsessive–compulsive personality disorder

B Avoidant personality disorder

C Schizoid personality disorder

D Dependent personality disorder

E Passive–aggressive personality disorder

64. The most useful treatment for intractable post-lumbar puncture headache is:

A Repeat lumbar puncture with blood patch

B Bed-rest for 2 weeks

C Acetaminophen

D Hydrocodone

E Sumatriptan

65. Which one of the following baseline lab values would be the least important to obtain on a patient starting lithium therapy:

A Thyroid function tests

B Electrocardiogram

C White blood cell count

D Renal function tests

E VDRL

66. Compression of which one of the following peripheral nerves results in meralgia paresthetica?

A Sciatic

B Lateral femoral cutaneous

C Obturator

D Superior gluteal

E Common peroneal

67. Following a long day in the hospital, you visit your best friend from college who has recently had a baby. The child has been spending more of her time asleep than awake and is not particularly aware of the environment. If your friend were to ask you which of Mahler’s stages of separation–individuation the child fits into, you would confidently answer:

A Normal autism

B Symbiosis

C Object constancy

D Practicing

E Differentiation

68. The classic finding on needle electromyography (EMG) that denotes the presence of a radiculopathy is:

A High-frequency, short-duration potentials

B Positive sharp waves and fibrillations

C Fasciculations

D Small, short motor unit potentials

E Myotonia

69. Which one of the following brain structure’s dopaminergic neurons have been linked with Tourette’s syndrome and the development of tics?

A Caudate

B Substantia nigra

C Amygdaloid body

D Frontal cortex

E Hippocampus

70. A 35-year-old woman delivers her baby at 40 weeks gestation without complication. Seven days later she experiences an acute onset of pancephalic headache, behavioral and personality changes, irritability, intermittent seizures and diplopia. The most likely diagnosis of her problem is:

A Aseptic meningitis

B Cerebral venous thrombosis

C Complicated migraine

D Bacterial meningoencephalitis

E Pseudotumor cerebri

71. When should advanced directives be discussed with a patient?

A At a time when the patient is competent

B When admitted to the hospital

C When a question of capacity arises

D When the patient is in pre-op

E When in the outpatient office

72. The most common congenital viral infection in newborns is caused by which one of the followed pathogens?

A Cytomegalovirus (CMV)

B Herpes simplex (HSV)

C Human immunodeficiency virus (HIV)

D Rubella

E Measles

73. A pervasive and excessive need to be taken care of that leads to submissive and clinging behavior as well as fears of separation, is characteristic of which one of the following?

A Obsessive–compulsive personality disorder

B Avoidant personality disorder

C Schizoid personality disorder

D Dependent personality disorder

E Passive–aggressive personality disorder

74. The most frequent myopathy in patients over 50 years of age is:

A Dermatomyositis

B Polymyositis

C Facioscapulohumeral dystrophy

D Inclusion body myositis

E Oculopharyngeal muscular dystrophy

75. Which one of the following is a behavioral effect of opioids?

A Miosis

B Increased arousal

C Euphoria

D Diarrhea

E Tachycardia

76. Duchenne’s and Becker’s muscular dystrophies are both disorders linked to an absence or deficiency of which one of the following muscle membrane proteins?

A Dystrophin

B Laminin

C Dystroglycan

D Spectrin

E Merosin

77. A 60-year-old woman is 63 pounds (28 kg) overweight. She comes to her psychiatrist’s office with a complaint of increased irritability, noting a fight with her husband over how much sugar he put in her coffee one morning. She is fatigued and naps several times each day. She has no history of psychiatric problems, but adds that her husband now sleeps in the living room because of her snoring. Which one of the following is the most likely cause of the patient’s symptoms?

A Night terrors

B Major depressive disorder

C Bipolar disorder

D Narcolepsy

E Sleep apnea

78. The rate-limiting enzyme in the dopamine synthetic pathway is:

A Dopa decarboxylase

B Tyrosine hydroxylase

C Dopamine β-hydroxylase

D Phenylethanolamine N-methyltransferase (PNMT)

E Catechol-O-methyltransferase

79. Which one of the following is the therapeutic range for lithium?

A 6–12 mEq/L

B 0.5–1.9 mEq/L

C 0.6–1.2 mEq/L

D 2–4 mEq/L

E >4 mEq/L

80. Alcohol, benzodiazepine sedative-hypnotic agents, and barbiturates all predominantly exert their clinical effects on the brain at which one of the following receptor sites?

A Cholinergic nicotinic

B N-methyl-D-aspartate (NMDA)

C Glycine

D GABA-A

E GABA-B

81. An elderly man comes into his doctor’s office with symptoms of dementia and notable loss of executive functioning. Dysfunction of which one of the following brain regions would be most closely associated with the patient’s loss of executive functioning?

A Caudate nucleus

B Putamen

C Globus pallidus

D Frontal lobes

E Temporal lobes

82. A patient who is not malingering, but is believed to be producing the symptoms and signs of confusion or dementia involuntarily or unconsciously and believes that the symptoms are real is thought to have which one of the following conditions?

A Conversion disorder

B Ganser’s syndrome

C Capgras syndrome

D Hypochondriasis

E Folie-à-deux

83. A child in school fails test after test. No matter how hard he studies, he fails. Over time he views himself as destined to fail and stops trying. Which one of the following theories best applies to this child’s situation?

A The epigenetic principle

B Industry theory

C Cognition theory

D Learned helplessness

E Sensory deprivation

84. Entacapone and tolcapone exert their antiparkinsonian effects on which one of the following enzymes?

A Catechol-O-methyltransferase

B Monamine oxidase type A

C Monoamine oxidase type B

D Dopa decarboxylase

E Dopamine β-hydroxylase

85. Which one of the following could drastically increase lithium levels?

A Citalopram

B Carbamazepine

C Sertraline

D Ibuprofen

E Acetaminophen

86. The neurologic examination finding of Argyll Robertson pupils occurs in a majority of patients with which one of the following conditions:

A Lyme disease

B Multiple sclerosis

C Tabes dorsalis

D Bubonic plague

E Intracerebral aneurysm

87. A patient on paroxetine complains of nausea, insomnia, muscle aches, anxiety, and dizziness. Which one of the following is the most likely explanation?

A He has the flu

B He has irritable bowel syndrome

C He stopped taking his antidepressant abruptly

D He is faking the symptoms

E He has multiple sclerosis

88. A 33-year-old man with known epilepsy has a 45-second generalized tonic–clonic seizure at a bus stop outdoors. He is brought to the nearest ER and once he is arousable and awake is found to have a marked right-sided hemiparesis. What is the best explanation of this occurrence?

A Postictal stroke

B Todd’s paralysis

C Complicated postictal migraine

D Conversion disorder

E Transient ischemic attack (TIA)

89. Which one of the following does not have active metabolites?

A Oxazepam

B Chlordiazepoxide

C Triazolam

D Clonazepam

E Quazepam

90. The disease known as tropical spastic paraparesis is considered to be a _____ caused by the _____ virus.

A Myelopathy; HTLV-1

B Neuropathy; HTLV-1

C Myelopathy; herpes simplex

D Neuropathy; herpes simplex

E Neuropathy; HIV type 1

91. To control aggression in a mentally retarded child, which one of the following would be most effective?

A Clonazepam

B Mirtazapine

C Doxepin

D Lithium

E Naltrexone

92. Which one of the following diseases is not associated with expansion of trinucleotide repeat sequences?

A Friedrich’s ataxia

B Myotonic dystrophy

C Multiple system atrophy

D Fragile X syndrome

E Huntington’s disease

93. Which one of the following dopaminergic pathways or areas is associated with the antipsychotic effects of the antipsychotic medications?

A Nigrostriatal pathway

B Mesolimbic–mesocortical pathway

C Tuberoinfundibular pathway

D Caudate nucleus

E Amygdaloid body

94. The characteristic bedside neurologic diagnostic sign known as the “battle sign” is indicative of which one of the following pathologies:

A Basilar skull fracture

B Frontal lobe damage

C Increased intracranial pressure

D Hypocalcemia

E Impending transtentorial cerebral herniation

95. Of the following combinations of medications, which one would the knowledgeable psychiatrist most want to avoid?

A Fluoxetine–lithium

B Fluoxetine–phenelzine

C Citalopram–valproic acid

D Citalopram–aripiprazole

E Mirtazapine–lamotrigine

96. Which one of the following agents is the best choice for treating attention deficit–hyperactivity disorder in a patient with Tourette’s syndrome?

A Bupropion

B Methylphenidate

C Dextroamphetamine

D Atomoxetine

E Clonidine

97. A method of obtaining a prediction for the value of one variable in relation to another variable is called:

A Correlation coefficient

B Control group

C Analysis of variance (ANOVA)

D Regression analysis

E Null hypothesis

98. Which one of the following oral agents would be most beneficial in the treatment of limb spasticity related to multiple sclerosis?

A Clonazepam

B Phenytoin

C Lioresal

D Phenobarbital

E Cyproheptadine

99. A measurement of the direction and strength of the relationship between two variables is called:

A Correlation coefficient

B Control group

C Analysis of variance (ANOVA)

D Regression analysis

E Null hypothesis

100. A patient in the intensive care unit is delirious and agitated. The electroencephalogram demonstrates a characteristic pattern of triphasic waves and some generalized background slowing. What other clinical bedside sign would you be most likely to see in this patient on physical examination?

A Herpetic skin vesicles

B Penile chancre

C Asterixis

D Dupuytren’s contractures

E Pulmonary rales

101. A researcher at a university hospital takes rats and randomly crowds them, shocks them, feeds them at different times, and uses bright lights and loud noise to interrupt their sleep. The rats eventually decrease their movement and exploratory behavior. What can be learned from this?

A This researcher has serious psychological problems and really needs a girlfriend

B This is evidence for Freud’s theory of coercive electrocution

C Unpredictability and lack of environmental control play a large role in the generation of stress

D Rats enjoy loud noise and bright lights, especially when accompanied by electric shocks

E Aggression among members of the same species is common

102. Which one of the following is not a potential side effect of valproic acid?

A Weight gain

B Alopecia

C Hemorrhagic pancreatitis

D Thrombocytosis

E Liver failure

103. A patient is admitted to the hospital for suicidal behavior. The insurance company refuses to pay for the stay after the third day, and the patient is still suicidal. Which one of the following answer choices is the most ethical way to proceed?

A Send the patient home because the insurance company refuses to pay

B Secretly tell the patient to sign out against medical advice

C Make the patient pay out of pocket, and if they can’t afford it, discharge them

D Sue the insurance company

E Continue to treat the patient as long as necessary, and file an appeal with the insurance company after discharge

104. The mechanism by which clonidine can help alleviate the symptoms of opioid withdrawal is through mediation of:

A Norepinephrine reuptake inhibition

B Alpha-2 adrenergic agonism

C Alpha-1 adrenergic antagonism

D Dopamine antagonism

E Serotonin antagonism

105. On a urine toxicology screen, how long can heroin be detected?

A 12 hours

B 48 hours

C 4 weeks

D 72 hours

E 8 days

106. An elderly hospitalized patient with vascular risk factors has a stroke. The patient’s behavior following the stroke is noted to be unusually calm and markedly hypersexual. This presentation likely resulted from a stroke to the:

A Hippocampi

B Nucleus accumbens

C Hypothalamus

D Occipital lobes

E Amygdaloid bodies

107. A psychiatrist is working on an inpatient unit and one of her patients has a very resistant case of depression. She considers the option of giving the patient electroconvulsive therapy (ECT). Which one of the following should make her most concerned about giving this patient ECT?

A Pregnancy

B Past seizures

C Family history of severe depression

D Psychotic symptoms

E Recent myocardial infarction

108. A 20-year-old man comes to your office with his mother because of behavioral problems. On examination you note that he is verbally inappropriate, mildly mentally retarded, very tall and somewhat obese and has a small penis and scrotum. His condition is most likely due to which one of the following?

A Absence of an X chromosome (XO)

B Presence of an extra X chromosome (XXY)

C Trisomy 21

D Deletion on the paternal chromosome 15

E Trisomy 18

109. A psychiatrist is consulted on a medical unit because there is a patient with a substance abuse history who is in need of pain control. Which one of the following answer choices would be the best way to treat this patient’s pain?

A Large doses of opiates

B A mixture of opiates and benzodiazepines

C Patient-controlled analgesia

D No opiates of any kind

E PRN (as needed) buprenorphine

110. A 35-year-old woman presents to your office with her husband. He tells you that she has experienced discrete episodes of physical and verbal aggression directed toward other people and property for the past year. Her husband states that the aggressiveness is not precipitated by any particular trigger and is completely unpredictable and intermittent. In between these bouts of aggression, the patient is otherwise fine and leads a normal life as a wife and mother. The most plausible diagnosis is:

A Borderline personality disorder

B Intermittent explosive disorder

C Antisocial personality disorder

D Psychotic disorder due to temporal lobe seizures

E Bipolar I disorder, with manic and psychotic episodes

111. While working on the ward of a state hospital, a psychiatrist comes across a patient with catatonic schizophrenia. The patient sits in one spot for extended periods of time, without changing position. This phenomenon is best described as:

A Psychomotor retardation

B Catalepsy

C Cataplexy

D Catatonia

E Stereotypy

112. Which one of the following agents has the unique mechanism of action of being a dopamine and norepinephrine reuptake inhibitor?

A Tiagabine

B Venlafaxine

C Duloxetine

D Bupropion

E Atomoxetine

113. Harry Harlow conducted a series of experiments with Rhesus monkeys. Some of his monkeys would stare vacantly into space, engage in self mutilation, and follow stereotyped behavior patterns. To which one of the following groups did these monkeys belong?

A Total isolation (no caretaker or peer bond)

B Mother-only reared

C Peer-only reared

D Partial isolation (can see, hear and smell other monkeys)

E Separation (taken from caretaker after developing bond)

114. Sildenafil, vardenafil and tadalafil all improve erectile dysfunction by which one of the following mechanisms?

A Phosphodiesterase 5 inhibition

B Calcium channel antagonism

C Nitric oxide antagonism

D Alpha 1 adrenergic antagonism

E Carbonic anhydrase inhibition

115. Which one of the following medications is the most likely to cause parkinsonian symptoms?

A Maprotiline

B Amoxapine

C Venlafaxine

D Doxepin

E Clomipramine

116. If you divide the incidence of a disease in those with risk factors by the incidence of the same disease in those without risk factors, the result is called the:

A Relative incidence

B Attributable risk

C Relative risk

D Period incidence

E Incidence risk

117. Cingulotomy is a treatment used for which one of the following choices?

A Psychosis

B Depression

C Obsessive–compulsive disorder

D Generalized anxiety disorder

E Pedophilia

118. Which one of the following causative etiologic factors is not believed to contribute to the genesis of dissociative identity disorder?

A A traumatic life event

B A vulnerability for the disorder

C Environmental factors

D Absence of external support

E Prior viral infection or exposure

119. Which one of the following is considered first-line treatment for a mute catatonic patient who is brought into the emergency room?

A Haloperidol

B Methylphenidate

C Risperidone

D Lorazepam

E Paroxetine

120. Which one of the following is not a feature of malingering?

A Findings are compatible with self-inflicted injuries

B Medical records may have been tampered with or altered

C Family members are able to verify the consistency of symptoms

D Symptoms are vague or ill defined

E History and examination do not yield complaints or problems

121. Which one of the following answer choices is most true concerning mutism?

A It is a psychiatric disorder only

B It is a neurological disorder only

C It is a function of a high-energy environment

D It is associated with both psychiatric and neurological conditions

E It is most frequently the result of head trauma

122. Therapy that is focused on the measurement of autonomic processes and teaching patients to gain voluntary control over these physiological parameters through operant conditioning is called:

A Stimulus-response therapy

B Biofeedback

C Relaxation training

D Behavior therapy

E Desensitization

123. A patient has overdosed on lithium following a fight with her boyfriend. Her lithium level is 2.8 mEq/L. She is exhibiting severe signs of lithium toxicity. Which one of the following answer choices is the best treatment at this time?

A IV fluids

B Celecoxib

C Wait and watch

D Hemodialysis

E Gastric lavage

124. The most useful long-term treatment parameter for a noncompliant patient with schizophrenia and history of violence would be:

A Long-term state psychiatric hospitalization

B Partial hospitalization

C Day treatment program

D Outpatient commitment program

E Social skills training

125. Which one of the following is a dangerous combination?

A MAOI–lorazepam

B MAOI–acetaminophen

C MAOI–meperidine

D MAOI–ziprasidone

E MAOI–loxapine

126. The psychiatrist’s right to maintain a patient’s secrecy in the face of a subpoena is known as:

A Privilege

B Confidentiality

C Communication rights

D Private rights

E Clinical responsibility

127. Which one of the following is not a requirement for treatment with clozapine?

A Baseline white blood cell (WBC) count before starting treatment

B Two WBC counts during the first 7 days of treatment

C Weekly WBC count during the first 6 months of treatment

D WBC counts every 2 weeks after the first 6 months of treatment

E WBC count every week for 4 weeks following discontinuation of clozapine

128. Which one of the following is not a change in sleep architecture noted in patients over age 65 years of age?

A Lower percentage of stages 3 and 4 sleep

B Less total REM sleep

C Fewer REM episodes

D Increased awakening after sleep onset

E Shorter REM episodes

129. Which one of the following will increase clozapine levels?

A Red wine

B Cimetidine

C Cheese

D Acetaminophen

E Aripiprazole

130. The most widely abused recreational substance among US high school students is:

A Alcohol

B Cocaine

C Lysergic acid diethylamide (LSD)

D Inhalants

E Cannabis

131. Beck, in the theory supporting his cognitive triad, felt that “distorted negative thoughts” lead to:

A Failure of good enough mothering

B Transitional object development

C Mania

D Depression

E Aggression toward the primary caregiver

132. The mechanism of action of the sleeping aid ramelteon involves which one of the following receptor systems?

A Acetylcholine

B GABA-A

C Histamine

D Melatonin

E Norepinephrine

133. What is alogia?

A Poverty of movement

B Poverty of emotion

C Poverty of speech only

D Poverty of thought content only

E Poverty of speech and thought content

134. The rapid-cycling specifier in bipolar I disorder applies to patients who have had _____ mood disturbance episodes over the previous _____.

A 4; 6 months

B 4; 12 months

C 6; 24 months

D 6; 12 months

E 3; 12 months

135. Which one of the following best explains pathological gambling?

A Primary reinforcement

B Random reinforcement

C Poor response to dexamethasone suppression testing

D Continuous reinforcement

E Cerebellar dysfunction

136. Late-onset schizophrenia is characterized by a more favorable prognosis and the onset of symptoms after age:

A 40 years

B 45 years

C 50 years

D 55 years

E 60 years

137. A group that does not receive treatment and is the standard for comparison is called the:

A Correlation coefficient

B Control group

C Analysis of variance (ANOVA)

D Regression analysis

E Null hypothesis

138. Bowlby’s stages of childhood attachment disorder, after a lengthy departure of the child’s mother, do not include which one of the following?

A Protest

B Despair

C Detachment

D Denial of affection

E Acceptance

139. You have a patient whom you think has both depression and attention-deficit/hyperactivity disorder. Which one of the following medications would be the best choice for this patient?

A Fluoxetine

B Paroxetine

C Bupropion

D Venlafaxine

E Imipramine

140. Which one of the following agents is FDA approved for use in the pediatric population under 18 years of age?

A Paroxetine

B Citalopram

C Sertraline

D Mirtazapine

E Venlafaxine

141. During a session a therapist tells the patient “I know you feel terrible right now, but things are going to get better with the passage of time”. This type of statement is characteristic of which type of therapy?

A Supportive psychotherapy

B Psychodynamic psychotherapy

C Psychoanalysis

D Play therapy

E Cognitive behavioral therapy

142. Which one of the following statements regarding risperidone intramuscular injection (Risperdal Consta) is false?

A The formulation comes in three doses: 25, 37.5 and 50 mg

B The drug must be refrigerated before reconstitution

C The drug must be reconstituted with sterile water

D The drug must be administered only to the deltoid or the gluteus muscles

E The drug is dosed to be administered every 2 weeks

143. A patient is brought into the emergency room unconscious with signs of respiratory depression. The patient was found unconscious on the bathroom floor and has written a suicide note saying that he wanted to die. An empty pill bottle that had contained his mother’s prescription for morphine was found on the bathroom floor. Which one of the following would the knowledgeable physician use to treat this patient?

A Buprenorphine

B Benztropine

C Naloxone

D Naltrexone

E Bromocriptine

144. Which one of KüblerRoss’ stages of reaction to impending death corresponds to a period when a patient goes through self-blame for his illness and asks “Why me?”

A Shock and denial

B Anger

C Bargaining

D Depression

E Acceptance

145. Asking patients if they are suicidal will:

A Increase the chance that they will kill themselves

B Help them plan out their suicide

C Scare the patients

D Have no influence on whether patients will attempt suicide

E Make patients refuse to speak to their therapist any further

146. The usual and accepted length of a period of grief following the death of a loved one can last up to:

A 3 months

B 6 months

C 9 months

D 12 months

E 24 months

147. In which one of the following groups has suicide increased dramatically over the past 40+ years?

A Geriatrics

B Married men

C Married women

D Adolescents

E Chronic alcoholics

148. Which one of the following sleep-promoting agents has the longest half-life?

A Ramelteon

B Zolpidem

C Zaleplon

D Eszopiclone

E Triazolam

149. What is the therapeutic range for valproic acid?

A 50–150 ng/mL

B 25–50 ng/mL

C 200–250 ng/mL

D 1000–1500 ng/mL

E 0.5–0.15 ng/mL

150. A psychiatrist interviews a Japanese immigrant who was brought to the hospital by her family for depression. In the meeting with her she does not endorse any significant symptoms. When speaking to the family, they state that she has been having many of the symptoms that she previously denied. Which one of the following is the most likely explanation for this situation?

A She is lying

B She is psychotic and paranoid

C Culture

D Mental retardation

E The family is lying

Answer Key – Test Number Three

1. C

2. A

3. B

4. B

5. E

6. D

7. E

8. E

9. D

10. C

11. D

12. A

13. E

14. E

15. E

16. A

17. D

18. B

19. E

20. D

21. C

22. B

23. E

24. B

25. B

26. E

27. C

28. B

29. B

30. C

31. D

32. D

33. D

34. B

35. D

36. D

37. B

38. B

39. B

40. B

41. D

42. D

43. E

44. A

45. E

46. B

47. D

48. B

49. C

50. A

51. E

52. E

53. C

54. A

55. C

56. D

57. D

58. C

59. A

60. D

61. C

62. D

63. B

64. A

65. E

66. B

67. A

68. B

69. A

70. B

71. A

72. A

73. D

74. D

75. C

76. A

77. E

78. B

79. C

80. D

81. D

82. B

83. D

84. A

85. D

86. C

87. C

88. B

89. A

90. A

91. D

92. C

93. B

94. A

95. B

96. D

97. D

98. C

99. A

100. C

101. C

102. D

103. E

104. B

105. D

106. E

107. E

108. B

109. C

110. B

111. B

112. D

113. D

114. A

115. B

116. C

117. C

118. E

119. D

120. C

121. D

122. B

123. D

124. D

125. C

126. A

127. B

128. C

129. B

130. E

131. D

132. D

133. E

134. B

135. B

136. B

137. B

138. E

139. C

140. C

141. A

142. D

143. C

144. B

145. D

146. D

147. D

148. D

149. A

150. C

Explanations – Test Number Three

Question 1. C. This question focuses on Piaget’s stages of cognitive development. (Just in case you don’t have it mastered after Test Two.) In the concrete operations stage, egocentric thought changes to operational thought where another’s point of view can be taken into consideration. In concrete operations children can put things in order and group objects according to common characteristics. They develop the understanding of conservation (a tall cup and a wide cup can both hold equal volumes of water) and reversibility (ice can change to water and back to ice again).

Human Development

K&S Ch. 2

Question 2. A. Transient global amnesia (TGA) presents with a reversible anterograde and retrograde memory loss. It is accompanied by inability to learn newly acquired information and total amnesia of events occurring during the attacks. Patients remain awake and alert during attacks, without motor or sensory deficits. Patients retain their personal information and identity and can carry on personal activities as usual. The patient may ask the same question repeatedly. Affected patients are usually in their fifties or older. Men are more often affected than women. Attacks are acute in onset and can last for several hours, but rarely greater than 12 hours. The true mechanism of TGA is unknown. One potential theory is a relationship between TGA and bilateral hippocampal ischemia, possibly migrainous in nature. Other theories postulate a relationship of TGA to epilepsy, migraine, brain tumor, and cerebrovascular events or risk factors. Onset often occurs after physical exertion, sexual exertion or exposure to extremes of temperature. The prognosis of patients with TGA is generally benign. There is no noted increased risk for future ischemic attacks or stroke. Recurrence is uncommon and there is no need for extensive workup. No particular treatment is indicated.

Neurology

B&D Ch. 6

Question 3. B. GABA is the major inhibitory neurotransmitter of the CNS. It is thought to decrease seizure activity, decrease mania, and lessen anxiety. It has three receptors, GABA-A, GABA-B, and GABA-C. Topiramate works on the GABA-A receptor, potentiating its activity. Gabapentin decreases seizure activity, but does not work directly on the GABA receptors or transporter. Tiagabine is an anticonvulsant which works by blocking the GABA transporter.

Psychopharmacology

K&S Ch. 3

Question 4. B. The case described in this question is a classic presentation of Lyme disease. The offending organism in Lyme infection is the spirochete Borrelia burgdorferi, whose vector is the deer tick: Ixodes dammini in the eastern USA, I. pacificus in the western USA and I. ricinus in Europe. Lyme is the great mimicker of other neurologic conditions and it can manifest in many different ways. Early infection can appear as a meningitis, a unilateral or bilateral Bell’s palsy (as in this question), a painful radiculoneuritis, optic neuritis, mononeuritis multiplex, or Guillain–Barré syndrome. After initial infection, in about two-thirds of patients a classic Lyme rash can be noted (erythema chronicum migrans), a painless expanding macular patch. Diagnosis begins with initial ELISA serologic screening, which, if positive, can be confirmed by Western blot testing. Cerebrospinal fluid lyme antibody can also be titered by polymerase chain reaction (PCR). Treatments of first choice are parenteral antibiotics: ceftriaxone or penicillin intravenously for 2–4 weeks. Tetracycline and chloramphenicol are treatment alternatives in penicillin or cephalosporin allergic patients. Cerebrospinal fluid persistence of antibody production may continue for years after successful treatment and remission and in isolation does not indicate active disease.

Rocky Mountain spotted fever is caused by the tick-borne spirochete Rickettsia rickettsii. Different tick species can carry this organism and the disease can be seen all over the world. The condition starts with fever, headache, muscle aches and gastrointestinal symptoms about 2–14 days after the tick bite. There is a rash that appears initially around the wrists and hands and spreads over days to the feet and forearms. Systemic symptoms can also appear and include meningoencephalitis, renal failure and pulmonary edema. Retinal vasculitis may be seen on funduscopic examination. Diagnosis is confirmed by direct immunofluorescence or immunoperoxidase skin biopsy staining. Treatment is undertaken with oral or parenteral tetracycline or chloramphenicol and a switch to oral doxycycline for a total of about 10–14 days of therapy.

Yersinia pestis is an organism causing the plague, a zoonotic infection of wild rodents, transmitted by the bites of infected fleas to human victims. Human infection can result in infectious lymphadenitis (bubonic plague), pneumonic, septicemic, or meningeal plague. Primary or secondary meningitis can occur from Y. pestis infection. Diagnosis is confirmed by cerebrospinal fluid Gram stain and culture. Treatment of primary infection is undertaken with intramuscular streptomycin twice daily for ten days. Meningitis is treated with intravenous chloramphenicol for at least ten days.

Leptospirosis is caused by zoonotic infection from Leptospira interrogans. This spirochete is transmitted to humans by contact with urine from infected rodents or farm animals or soil or water that contains the infected urine. About 15% of patients develop a meningitic picture. Severe systemic symptoms include jaundice, hemorrhage and renal failure. Diagnosis can be confirmed by organism isolation from blood or cerebrospinal fluid. Severe disease is treated with at least a week-long course of parenteral penicillin G. Less severe illness can be treated with a week-long course of oral doxycycline.

Treponema pallidum is the spirochete associated with syphilis, which is explained in depth elsewhere in this volume.

Neurology

B&D Ch. 53

Question 5. E. This is a clear case of body dysmorphic disorder. It is characterized by an imagined belief that there is a defect in part or all of the body. The belief does not approach delusional proportions. The patient complains of the defect and his perception is out of proportion to any minor physical abnormality that exists. The person’s concern is grossly excessive, but when pressed he can admit that it may be excessive (this is why it is not delusional). Treatment with serotonergic drugs and therapy is usually helpful; plastic surgery is not.

Somatic Symptom Disorders

K&S Ch. 17

Question 6. D. Tourette’s syndrome (TS) is a tic disorder that occurs in about 10 to 700 in 100 000 individuals. Boys are affected more often than girls. The manifestations begin between 2 and 10 years of age. Early signs include cranial motor tics, including eye blinks, stretching of the lower face and head-shaking. Vocal tics can include throat clearing, grunting, coughing, sniffing and involuntary swearing (coprolalia). Both motor and vocal tics must be present to meet criteria for the disorder. Symptoms tend to peak in severity during adolescence and often wane during adulthood. Behavioral manifestations are common in TS and can include attention-deficit/hyperactivity disorder (ADHD), obsessive–compulsive disorder, and conduct disorder. Obsessions and rituals often revolve around counting and symmetry. The diagnosis is clinical and is based on the appropriate history and physical examination.

Evidence suggests that TS is hereditary and likely follows an autosomal dominant pattern of inheritance. Concordance in monozygotic twins is greater than 85%. Exact mendelian inheritance pattern for the disorder is not yet elucidated. Positron emission tomography studies suggest increased dopaminergic activity in the ventral striatum and abnormal release and reuptake of dopamine as being probable pathophysiologic mechanisms.

Treatment of TS is symptomatic. Tics can be treated with conventional or atypical neuroleptic agents, such as haloperidol, pimozide, fluphenazine, risperidone, quetiapine, and olanzapine. Guanfacine and clonidine are useful for both tics and ADHD symptoms. Obsessive–compulsive disorder can be treated with the selective serotonin reuptake inhibitors. The stimulants methylphenidate, pemoline, and dextroamphetamine, must be used carefully as they can either alleviate or exacerbate tics. Atomoxetine, a non-stimulant ADHD medication that is a norepinephrine reuptake inhibitor, is an excellent choice for ADHD in TS because it does not exacerbate tics.

Neurology

B&D Ch. 71

Question 7. E. In order to diagnose schizoaffective disorder (as opposed to bipolar), one needs a two-week period where the patient has had psychotic symptoms in the absence of mood symptoms. The patient must also have had periods of mania or depression during which psychotic symptoms are present. A bipolar patient may have psychotic symptoms during periods of mania or depression, but these psychotic symptoms cease when the mood disturbance resolves.

Psychotic Disorders

K&S Ch. 14

Question 8. E. The safest choice of all the answers is of course electroconvulsive shock therapy (ECT). There is no real contraindication to ECT in the normal pregnancy. If the pregnancy is high-risk or complicated, fetal monitoring can be carried out during the procedure. Haloperidol would be the next best choice and is of course a butyrophenone antipsychotic agent. Haloperidol can pass into breast milk and so mothers should not breast feed if they are taking this drug. Haloperidol is a category C agent in pregnancy and it has been shown to cause teratogenicity in animals. Human studies are inadequate in this regard and the benefit needs to outweigh the risk before the drug is given to a pregnant patient. The first trimester is the most vulnerable period of pregnancy for teratogenic fetal effects. Lithium is of course contraindicated in pregnancy because of the risk of Ebstein’s anomaly of the tricuspid valves. The risk of Ebstein’s anomaly is 1 in 1000. Lithium is also excreted into breast milk. Lithium is a category D agent in pregnancy. Divalproex sodium is also dangerous in pregnancy because of the first trimester risk of fetal spina bifida and neural tube defects in about 1–2% of those taking the drug in their first trimester. Folic acid supplementation (1–4 mg daily) taken during the first trimester of pregnancy reduces the risk of neural tube defects with divalproex sodium. Divalproex is a category D agent in pregnancy. Aripiprazole has not been well studied in pregnancy, but is a category C agent in pregnancy. There are animal studies that have revealed fetal abnormalities as a direct result of maternal exposure to aripiprazole.

Management in Psychiatry

K&S Ch. 36

Question 9. D. This is a case of delirium. Delirium presents as confusion, impaired consciousness (often fluctuating), emotional lability, hallucinations or illusions, and irrational behavior. Its onset is rapid and its course fluctuates. Thinking is often disordered, and attention and awareness are often impaired. Causes are usually medical or organic in nature (metabolic imbalance, infection, intracranial bleed, etc.). The patient’s increased age makes him more susceptible to a delirium. Dementia and depression are incorrect because the time of onset of symptoms for these disorders is over weeks to months, not days.

Neurocognitive Disorders

K&S Ch. 10

Question 10. C. Metoclopramide (Reglan) is a potent antiemetic agent that is a benzamide derivative, has phenothiazine-like properties, and can cause extrapyramidal side effects. It is a potent antagonist of dopamine type 2 (D2) receptors and blocks these receptors on the chemoreceptor trigger zone of the area postrema which prevents nausea and emesis. Because of its affinity for the D2 receptor it has been known to cause extrapyramidal side effects (EPS), especially at higher doses, and can also cause tardive dyskinesia after long-term use and discontinuation.

Ondansetron (Zofran) is a potent antiemetic like metoclopramide, but its mechanism does not involve D2 so it does not have the potential to cause EPS or tardive dyskinesia. It is a potent 5-HT3 antagonist and also works in the area postrema and likely on peripheral vagal nerve receptors.

Hydroxyzine (Atarax, Vistaril), like diphenhydramine (Benadryl) is an antihistamine that also has analgesic and antiemetic properties. Is has no effect on dopamine receptors and therefore does not cause EPS or tardive dyskinesia.

Tizanidine (Zanaflex) is a potent sedating muscle relaxant that works via α-2 adrenergic agonism. It can lower blood pressure, much like clonidine which has this same mechanism of action. It does not affect dopamine receptors and does not cause EPS or tardive dyskinesia.

Psychopharmacology

K&S Ch. 36

Question 11. D. Sleep attacks are the most common symptom of narcolepsy. Narcolepsy is also characterized by cataplexy (a sudden loss of muscle tone) and hallucinations while falling asleep and waking up. Patients with narcolepsy can have sleep paralysis where they wake up and are often unable to move. Their sleep also characteristically goes into REM cycles when sleep begins. Treatments involve taking regular naps during the day, stimulants, and antidepressants.

Sleep Wake Disorders

K&S Ch. 24

Question 12. A. The primary auditory cortex localizes to the superior temporal gyrus (Heschl’s gyrus) in both temporal lobes. Cortical deafness can result from bilateral strokes to the temporal lobes destroying Heschl’s gyri. The thalamus is of course the relay station for much of the sensory input to the brain. The frontal lobes are responsible for attention, concentration, set-shifting, organization and executive functioning and planning. The occipital lobes are the location of the calcarine cortex that is responsible for interpreting and processing visual input and stimuli.

Neurology

B&D Chs 11&12

Question 13. E. Glycine is a neurotransmitter synthesized from serine. It is a necessary adjunctive neurotransmitter at the NMDA receptor that binds with glutamate. It is also an independent inhibitory neurotransmitter with its own receptors that open chloride ion channels. The activity of glycine on the NMDA receptor is an area of research for schizophrenia, with some studies showing improvement in negative symptoms with the use of glycine or glycine analogues. The highest concentrations of glycine receptors have been found in the spinal cord. Mutations of this receptor lead to a rare condition called hyperekplexia where the main symptom is an exaggerated startle response.

Basic Neuroscience

K&S Ch. 3

Question 14. E. Gabapentin and pregabalin are both FDA-approved for the systemic oral treatment of postherpetic neuralgia. The lidocaine transdermal patch (Lidoderm) is FDA-approved for the topical and local treatment of postherpetic neuralgic pain. Carbamazepine and oxcarbazepine are not FDA-approved, but both have been shown to be acceptable alternatives for the treatment of trigeminal and postherpetic neuralgia. Phenytoin, an anticonvulsant agent, has no place in the treatment of postherpetic neuralgia.

Neurology

B&D Ch. 44&67

Question 15. E. This is a clear case of acute stress disorder. Acute stress disorder occurs after a person is exposed to a traumatic event. The patient then feels anxiety, detachment, derealization, feelings of being “in a daze”, dissociative amnesia, and numbing. Flashbacks and avoidance of stimuli can occur. The symptoms do not last longer than 4 weeks, and occur within 4 weeks of the traumatic event (as opposed to PTSD where symptoms must last more than 1 month).

Anxiety Disorders

K&S Ch. 16

Question 16. A. Epidural hematomas result most often from head trauma and skull fracture that cause a tear to the middle meningeal artery or one of its branches. They occur between the dura and the skull table and appear as a biconvex or lens-shaped hyperdensity on CT scan of the brain. Most epidural hematomas are localized to the temporal or parietal areas, but they can occur elsewhere in the brain as well.

Subdural hematoma is the most common intracranial hematoma, found in 20–25% of all traumatic brain injury patients who are comatose. The subdural hematoma is believed to result from tearing of the bridging veins over the cortical surface, or from trauma to the venous sinuses or their tributaries. Subdural hematoma is more common in patients over 60, particularly alcoholics, presumably because the dura is tightly adhered to the skull table in this population. The other answer choices are distracters.

Neurology

B&D Ch. 50

Question 17. D. Though one could argue for bits and pieces of other diagnoses, when taking the whole picture into consideration, this patient is a case of borderline personality disorder. Borderline personality disorder is characterized by frantic efforts to avoid abandonment, unstable interpersonal relationships, disturbed self image, impulsive behavior, recurrent suicidality or self mutilation, affective instability, chronic feelings of emptiness, intense anger, and stress related paranoia or severe dissociative symptoms. The woman in the question clearly presents with many of these symptoms.

Personality Disorders

K&S Ch. 27

Question 18. B. Electroencephalograms (EEG) of patients with Creutzfeldt–Jakob disease (CJD) classically demonstrate theta and delta waves with periodic burst-suppression pattern and periodic, rhythmic paroxysmal sharp-wave complexes. These high-amplitude sharp wave-forms are present in up to 80% of patients with CJD. This pattern can also be noted in subacute sclerosing panencephalitis, but with longer interburst intervals.

The 3-per-second spike and wave pattern is indicative of absence seizures (petit mal epilepsy). Temporal spikes would be indicative of a seizure focus in the temporal lobe, most often of the partial-complex variety. Generalized background slowing can be noted in postictal states, coma, delirium, or following anoxic brain injury. It usually indicates a decrease in the level of consciousness. Periodic lateralizing epileptiform discharges (PLEDS) are characteristic of herpes simplex encephalitis, but can also be seen in acute hemispheric stroke, tumors, abscesses and meningitis.

Neurology

B&D Chs 32&53

Question 19. E. The criteria for polysubstance dependence state that during a 12 month period the patient repeatedly used at least three groups of substances (excluding caffeine and nicotine), but no single substance predominated. During this period, the dependence criteria need to be met for groups of substances, but not for any one particular substance.

Substance Abuse and Addictive Disorders

K&S Ch. 12

Question 20. D. Aseptic or viral meningitis tends to produce a cerebrospinal fluid (CSF) assay with mild to moderate lymphocytic pleocytosis, normal glucose and normal to mildly elevated protein. Gram stain and cultures would be negative. A bacterial meningitis would produce a CSF as in answer choice C, with marked lymphocytosis, particularly polymorphonuclear neutrophils, markedly increased protein and decreased glucose. A fungal meningitis would produce a CSF as in answer choice B, with moderate lymphocytic pleocytosis, mildly decreased glucose and mildly increased protein. Tuberculous meningitis would produce a CSF with mild lymphocytic pleocytosis, mildly decreased glucose and markedly increased protein, as in answer choice A. Herpetic meningitis would produce a similar CSF picture to that in aseptic meningitis, but may also reveal a predominance of red blood cells in the CSF. Answer choice E is a CSF assay indicative of Guillain-Barré syndrome (acute inflammatory demyelinating polyneuropathy) which is characterized by a cytoalbuminergic dissociation with high protein (> 55 mg/dL) and an absence of significant pleocytosis.

Neurology

B&D Ch. 53

Question 21. C. This question focuses on Mahler’s stages of separation–individuation. The first stage is normal autism, lasting from birth to 2 months. In this stage the baby spends more time asleep than awake. The next stage is symbiosis, from 2 months to 5 months. In this stage the baby is developing the ability to distinguish the inner from the outer world. The child perceives himself as being part of a single entity with his mother. The following stage is differentiation, from 5 to 10 months. Here the child is drawn further into the outside world and begins to distinguish himself from his mother. Next is practicing. Practicing is from 10 to 18 months and is characterized by the baby’s ability to move independently and explore the outside world. Practicing is followed by rapprochement between 18 and 24 months. In rapprochement the child’s independence vacillates with his need for his mother. The child moves away from the mother, then quickly returns for reassurance. Mahler’s last stage is object constancy, from 2 to 5 years. In this stage the child understands the permanence of other people, even when they are not present. It would be wise to know all of Mahler’s stages well. You never know where you might see them again!

Human Development

K&S Ch. 2

Question 22. B. Arsenic poisoning has both central and peripheral nervous system manifestations. Systemic manifestations include nausea, vomiting, diarrhea, hypotension, tachycardia, and vasomotor collapse that can lead to death. Stupor or encephalopathy may develop. Peripherally, arsenic causes a distal axonal neuropathy. Symptoms of the neuropathy may develop after two to three weeks following initial exposure. Skin manifestations may develop with more chronic exposure, particularly keratosis, melanosis, malignancies and characteristic Mees’ lines of the fingernails. Mees’ lines are white transverse striations of the fingernails that occur about 3 to 6 weeks after initial arsenic exposure. Mees’ lines can also be noted in thallium poisoning and from chemotherapy exposure.

image

Lead poisoning occurs in those who work with metal, soldering, battery manufacturing, and in smelting factories. Children tend to develop an acute encephalopathy and adults a polyneuropathy with lead poisoning. Children can get exposed to lead by ingestion of old paint that contains lead. They develop an acute gastrointestinal illness and ultimately behavioral manifestations with confusion, drowsiness, generalized seizures and intracranial hypertension. In adults, the lead neuropathy manifests predominantly as a motor neuropathy which presents as bilateral wrist drop and/or foot drop. A rare sign of lead poisoning is the appearance of a blue line at the gingival margin in patients with poor oral hygiene. Adults sometimes develop a gastrointestinal illness and a hypochromic, microcytic anemia. Lead encephalopathy is managed supportively. Systemic corticosteroids can be given to reduce brain edema and chelating agents like dimercaprol (British anti-Lewisite) are also prescribed.

Mercury poisoning can occur from mercury vapor inhalation in the making of batteries, electronics manufacturing and in the past in the hat-making industry. Clinical manifestations include personality changes (“mad as a hatter”), irritability, insomnia, drowsiness, confusion and stupor. Other systemic symptoms and signs include intention tremors (“hatter’s shakes”), proteinuria, glycosuria, hyperhidrosis, and muscle weakness. Chronic exposure can lead to visual field deficits, sensory disturbances, progressive ataxia, tremor, and cognitive impairment. Treatment using chelating agents does not always increase rate or extent of recovery.

Organophosphate poisoning can occur from exposure to pesticides, herbicides, and flame retardants. Organophosphates inhibit acetylcholinesterase by phosphorylation and result in cholinergic toxicity. Clinical manifestations include salivation, lacrimation, nausea, bronchospasm, headache, weakness and in severe cases, bradycardia, tremor, diarrhea, pulmonary edema, cyanosis, and convulsions. Coma can ensue and death can result from respiratory or cardiac failure. The treatment of choice is pralidoxime and atropine. Pralidoxime helps restore acetylcholinesterase and atropine helps to counteract muscarinic adverse effects.

Ionizing radiation is a risk factor for central nervous system tumor formation, particularly meningiomas and nerve sheath tumors. Electromagnetic radiation is a likely risk factor for both leukemia and brain tumor.

Neurology

B&D Chs 52&58

Question 23. E. This question covers some of the statistical terms that could be fair game for an exam. Learn them well. A control group is a group that does not receive treatment and is a standard of comparison. Analysis of variance is a set of statistical procedures which compares two groups and determines if the differences are due to experimental influence or chance. Regression analysis is a method of using data to predict the value of one variable in relation to another. The null hypothesis is the assumption that there are no differences between two samples of a population. When the null hypothesis is rejected, differences between the groups are not attributable to chance alone. Correlation coefficient will be covered in detail in future questions.

Statistics

K&S Ch. 4

Question 24. B. Organophosphates inhibit acetylcholinesterase and cause cholinergic toxicity. Organophosphates are found predominantly in pesticides and herbicides and poisoning generally occurs in agriculture workers who are exposed following spraying in fields. Pralidoxime and atropine are the agents of choice to treat organophosphate toxicity. The other answer choices are simply distracters. For a more detailed explanation of the organophosphates, see Question 22.

Neurology

B&D Ch. 58

Question 25. B. Emil Kraepelin used the term dementia praecox to describe schizophrenia. It referred to the early onset of memory loss or decreased cognitive function often seen in patients with schizophrenia. Patients with dementia praecox were found to have a long deteriorating course with hallucinations and delusions. It was Eugen Bleuler who coined the term schizophrenia. The term schizophrenia is often misconstrued to mean split personality, which in modern times is referred to as dissociative identity disorder. Other answer choices given are unrelated distracters.

History of Psychiatry

K&S Ch. 13

Question 26. E. Lead poisoning can cause a rare appearance of a blue line at the gingival margin in patients with poor oral hygiene. Lead can also cause a microcytic, hypochromic anemia. A more detailed explanation of lead poisoning appears in Question 22 above.

Manganese classically causes neurotoxicity after months or years of exposure. Manganese miners are particularly at risk due to their prolonged inhalation of the toxin. Parkinsonism and motor symptoms can develop from manganese poisoning and usually follow initial manifestations of behavioral changes, headache and cognitive disturbances (“manganese madness”). A characteristic gait, walking on the toes with spine erect and elbows flexed, called the “cock-walk,” can emerge. The condition is generally poorly-responsive to l-dopa therapy.

Thallium poisoning results in a severe neuropathy and central nervous system degeneration. A chronic, progressive sensory polyneuropathy can develop. Thallium causes potassium depletion which can result in cardiac abnormalities, such as sinus tachycardia, T-wave changes and U waves. Alopecia can develop two to four weeks after initial exposure. Treatment with intravenous potassium chloride and oral potassium ferric ferrocyanide, and hemodialysis and forced diuresis can help in recovery from acute thallium intoxication. Arsenic and mercury poisonings are discussed in more detail in Question 22.

Poisoning

B&D Ch. 58

Question 27. C. This is a case of shared psychotic disorder (folie-à-deux). In shared psychotic disorder, a delusion develops in an individual who is in a close relationship with someone who already has an established delusion. This delusion is similar to that of the person who already has the established delusion.

Psychotic Disorders

K&S Ch. 14

Question 28. B. Ataxia results from cerebellar lesions. Midline cerebellar vermian lesions cause truncal ataxia. Head tremor and truncal instability leading to oscillation of the head and trunk in a seated or standing posture (titubation) may result from lesions of the cerebellar vermis. In lateralized lesions to the cerebellar hemisphere, signs and symptoms occur ipsilateral to the lesion. Cerebellar hemispheric lesions would be expected to cause ipsilateral limb ataxia and/or dysmetria, either of the arm or leg, or both, depending on the location of the lesion. Dysdiadochokinesis can result from a cerebellar hemispheric lesion. This refers to a deficit in the ability to perform smooth rapid alternating movements of the hands or feet. A disturbance in both rhythm and amplitude of these alternating movements can be noted with cerebellar lesions.

Lesions arising in the cerebellopontine angle area can result in cranial neuropathy particularly to nerves V, VII and VIII. Manifestations include ipsilateral peripheral facial palsy (Bell’s palsy, VII nerve palsy), ipsilateral facial numbness and weakness of masseter muscles (V nerve palsy) and ipsilateral hearing loss, tinnitus and vertigo (VIII nerve palsy).

Thalamic lesions can cause any number of deficits. Lacunar thalamic infarcts can lead to a pure sensory stroke (contralateral to the lesion), or to a sensorimotor stroke if the lesion also invades the internal capsule. Thalamic lesions can cause a rare disorder of central thalamic pain (again contralateral to the lesion and usually in the extremities and/or the face) known as the thalamic syndrome of Dejerine–Roussy.

Strokes or lesions to the midbrain can cause a variety of symptoms and syndromes. One such classic presentation is the Parinaud’s syndrome which can result from a midbrain lesion arising from ischemia to the posterior cerebral artery (PCA) penetrating branches. This manifests with supranuclear paresis of eye elevation, eyelid retraction, skew deviation of the eyes, defective convergence and convergence-retraction nystagmus and light-near dissociation. Another midbrain stroke syndrome is Weber’s syndrome, also arising from ischemia in the PCA territory. There is a contralateral hemiplegia of the face, arm and leg and ipsilateral oculomotor (III nerve) paresis with a dilated fixed pupil.

Neurology

B&D Chs 20&51

Question 29. B. Here is another chance to understand glycine if you did not master it after Question 13. Glycine is a neurotransmitter synthesized from serine. It is a necessary adjunctive neurotransmitter at the NMDA receptor that binds with glutamate. It is also an independent inhibitory neurotransmitter with its own receptors that open chloride ion channels. The activity of glycine on the NMDA receptor is an area of research for schizophrenia, with some studies showing improvement in negative symptoms with the use of glycine or glycine analogs. The highest concentrations of glycine receptors have been found in the spinal cord. Mutations of this receptor lead to a rare condition called hyperekplexia where the main symptom is an exaggerated startle response.

Basic Neuroscience

K&S Ch. 3

Question 30. C. Suspected meningitis with obtundation is not a contraindication to performing a lumbar puncture (LP). All of the other answer choices are indeed absolute or relative contraindications to LP. A localized infection at the level of the puncture would be a contraindication to performing the procedure, due to a risk of the LP needle seeding the infection into a meningitis. Thrombocytopenia may lead to excessive and uncontrollable bleeding if LP is performed. Fresh frozen plasma or platelet transfusion may need to be given prior to the procedure, if the LP is deemed to be essential to further diagnosis and management of the patient. Cerebral mass lesion is generally an absolute contraindication to LP because of the risk of cerebral or cerebellar herniation following the procedure. Transtentorial herniation can arise if a LP suddenly releases elevated cerebrospinal fluid pressure forcing the medial temporal lobe downwards causing the midbrain or the cerebellum to compress the cervicomedullary junction through the foramen magnum. Papilledema is a sign of increased intracranial pressure and a possible mass lesion. Funduscopic examination must be conducted before LP is performed. Head trauma is also a contraindication to LP, because it too may lead to a herniation syndrome because of increased intracranial pressure.

Neurology

B&D Ch. 31

Question 31. D. In case Question 15 didn’t solidify acute stress disorder and post-traumatic stress disorder in your mind, take advantage of this question to clarify your understanding. Acute stress disorder occurs when a person is exposed to a traumatic event. The patient then feels anxiety, detachment, derealization, feelings of being “in a daze”, dissociative amnesia, and numbing. Flashbacks and avoidance of stimuli can occur. The symptoms do not last longer than 4 weeks, and occur within 4 weeks of the traumatic event (as opposed to PTSD where symptoms must last more than 1 month).

Anxiety Disorders

K&S Ch. 16

Question 32. D. Fragile X syndrome is believed to be the most frequent cause of mental retardation (MR) in the general population. The syndrome is characterized by moderate to severe MR, macroorchidism, prominent jaw, large ears, and jocular high-pitched speech. Hyperactivity and inattention are characteristic in affected males with Fragile X syndrome. The chromosomal anomaly lies at Xq28.

Down’s syndrome results generally from chromosomal nondisjunction that leads to a trisomy 21. This is most often due to advanced maternal age. Manifestations include infantile hypotonia, hyperlaxity of the joints, brachycephaly, flattened occiput, mental retardation, upslanting palpebral fissures, flattened nasal bridge, epicanthal folds, small ears, hypoplastic teeth, short neck, lenticular cataracts, speckling of the iris (Brushfield’s spots), brachydactyly, simian creases and congenital cardiac anomalies (in 30–40% of cases). Down’s syndrome patients acquire Alzheimer-like dementia much earlier than in the general population at large.

Trisomy 18 occurs in about 1 in 6000 live births. Fifty per cent of infants do not survive past the first week of life. Manifestations include low-set ears, small jaw, hypoplastic fingernails, mental retardation, cryptorchidism, congenital heart disease (patent ductus arteriosus, atrial septal defect, ventricular septal defect), microcephaly, and renal anomalies such as polycystic kidneys.

Cri-du-chat syndrome is a hereditary congenital mental retardation syndrome caused by a deletion at the short arm of chromosome 5p15.2. It occurs in 1 in 20 000 to 50 000 live births. Manifestations include severe mental retardation, microcephaly, round face, hypertelorism, micrognathia, epicanthal folds, hypotonia, and low-set ears. Newborns present with a cat-like high-pitched cry that is considered to be diagnostic of the disorder. A majority of these patients do not live past early childhood.

Prader–Willi syndrome is considered to be an autosomal dominant disorder resulting in a deletion to chromosome 15q11-13. Clinical stigmata include decreased fetal activity, obesity, mental retardation, hypotonia, short stature, hypogonadism, and small hands and feet. High caloric intake leads to diabetes and cardiac failure in many patients and many Prader–Willi syndrome patients do not survive past 25 to 30 years of age.

Neurology

B&D Ch. 61

Question 33. D. Dissociation is a disturbance in which a person fails to recall important information. There are a number of dissociative disorders including dissociative amnesia, dissociative fugue, and dissociative identity disorder. In all of these situations the patient’s lack of recall is in excess of what could be explained by ordinary forgetfulness. Used as a defense mechanism, the term dissociation represents an unconscious process involving the segregation of mental or behavioral processes from the rest of the person’s psychological activity. It can involve the separation of an idea from its emotional tone, as one sees in conversion disorder.

Dissociative Disorders

K&S Ch. 20

Question 34. B. Shaken-baby syndrome from child abuse can lead to death from intracranial, subarachnoid, and subdural hemorrhages. These result from vascular shearing and tearing due to the violent back-and-forth movement that results in cerebral trauma. Retinal hemorrhages are often noted as well in these cases. Hemorrhages can lead to seizures and ischemic cerebral infarction due to vasospasm of intracranial vessels. Ninety-five per cent of severe intracranial injuries in children 1 year of age or younger are due to child abuse. The other answer choices are of less pathological importance in cases of child abuse.

Neurology

B&D Ch. 51

Question 35. D. This is a case of normal bereavement. Crying, weight loss, decreased libido, withdrawal, insomnia, irritability, and poor concentration and attention can all be part of normal bereavement. Keys to normal bereavement are that suicidality is rare, it improves with social contacts, and it lacks global feelings of worthlessness. In depression, one finds anger and ambivalence towards the deceased, suicidality is common, and social contacts do not help, thus the person isolates. In addition others find the depressed person irritating or annoying, whereas the bereavement patient evokes sympathy from others. In depression the patient may feel that he or she is worthless, which is not the case in bereavement. With respect to the other answer choices, these symptoms aren’t going on long enough for dysthymic disorder. There are no anxiety symptoms described to argue for acute stress disorder. There is no mania described, and the patient does not meet criteria for major depression.

Depressive Disorders

K&S Ch. 2

Question 36. D. Lower motor neuron signs include hypotonia, muscle atrophy, fasciculations, hyporeflexia, flaccidity, muscle cramps, and marked motor weakness. These are considered to arise from lesions that are distal (i.e., peripheral) to the anterior horn cells where the upper and lower motor neurons synapse. Upper motor neuron signs include hyperreflexia, spasticity, Babinski’s sign, clonus, pseudobulbar palsy, loss of dexterity, and mild motor weakness. Both upper and lower motor neuron signs can be demonstrated in amyotrophic lateral sclerosis (Lou Gehrig’s disease), a disease which carries both sets of characteristics. Poliomyelitis is a classic disease of the lower motor neurons and carries the manifestations of lower motor neuron disease as noted above.

Neurology

B&D Ch. 76

Question 37. B. Mahler’s stages of separation–individuation are back again! This question describes symbiosis, which lasts from 2 months to 5 months. In this stage the baby is developing the ability to distinguish the inner from the outer world. The child perceives himself as being part of a single entity with his mother. For more details on Mahler, see Question 21.

Human Development

K&S Ch. 2

Question 38. B. Brandt–Daroff exercises can be extremely helpful in the treatment of benign paroxysmal positional vertigo (BPPV). The exercises involve rapidly lying down on one side from a seated position and remaining there for about 30 seconds before sitting up for 30 seconds and repeating the maneuver in the opposite direction. Patients are supposed to complete 20 full repetitions twice daily. Most patients see relief within a week, but it may take three months or more to achieve complete symptom remission. Once in remission, most patients are cured of the disorder.

Benign paroxysmal positional vertigo is a vestibulopathy that is believed to be a result of sludge deposition or otoliths in the utricle of the posterior semicircular canal of the inner ear. When the patient moves his head, the movement of the material in the semicircular canal irritates the hair cells in the inner ear and triggers a severe acute onset of rotational vertigo, with nausea, possible vomiting, and characteristic rotatory nystagmus. Diagnosis is clinical and is suggested by the history. Symptoms can usually be evoked acutely by the Dix–Hallpike maneuver in which the patient’s head is rapidly lowered to the bed by the examiner. The head is then turned with one ear down to the bed. The examiner notes the reproduction of vertigo and nystagmus, which are generally pathognomonic for BPPV. The fast phase of the nystagmus is noted in the direction of the lower ear (the ear with the vestibular problem) when the patient looks toward the affected side. In primary gaze, the fast rotational phase is vertical and upward, rotating toward the affected (lower) ear. Other treatments include meclizine, an antiemetic agent that helps with nausea and vomiting, but not generally with the vertigo. Metoclopramide (Reglan) is also an antiemetic agent that can help in a similar way to meclizine. Diazepam can lessen anxiety that can be associated with severe symptoms. Gabapentin is not indicated, nor is it particularly useful, in BPPV.

Neurology

B&D Chs 37

Question 39. B. All of this question’s answer choices are criteria for diagnosis of kleptomania, except that, before the theft, the patient has an increased sense of tension. Kleptomania is found within the larger heading of impulse control disorders. In kleptomania the patient engages in repeated stealing of objects that they do not need. An important part of the disorder is the sense of tension before the act and the sense of pleasure or relief afterward.

Disruptive, Impulse Control, Conduct Disorders, and ADHD

K&S Ch. 25

Question 40. B. The persistent vegetative state usually follows from a period of coma and is believed to signal the onset of severe cerebral cortical damage, often due to brain anoxia, trauma, or both. A period of one month of coma needs to elapse before the patient can be said to be in a persistent vegetative state. The condition is characterized by absence of cognitive function and absent awareness of the surrounding environment, despite a preserved sleep–wake cycle. Spontaneous movements can be noted and eye opening can be preserved, but the patient does not speak and cannot obey commands. Eye tracking and swallowing can also be preserved. There is no response to noxious stimulation in the persistent vegetative state. Permanent irreversible brain damage is believed to set in after about 12 months of a persistent vegetative state that follows brain trauma and usually after about 3 or more months following anoxic brain injury.

Neurology

B&D Ch. 5

Question 41. D. Ataque de nervios is a culture bound anxiety syndrome associated with those from Latin-American cultures. Its symptoms include headache, insomnia, anorexia, fear, anger, despair, and diarrhea.

The question says nothing about psychosis, so schizophrenia and schizoaffective disorder are incorrect. This case does not meet criteria for panic disorder. Myoclonic sleep disorder is not even remotely related to the symptoms given. Add in the fact that the patient is Hispanic, and the answer choice is clear, ataque de nervios.

Cultural Issues in Psychiatry

K&S Ch. 14

Question 42. D. Depression classically disrupts rapid eye movement (REM) sleep patterns. The most typical effect of depression on sleep however, is early morning awakening. Depression can also shorten REM latency (one hour or less). Depression can increase the total percentage of REM sleep and flip the predominance of REM from the early morning near awakening, to the beginning of the night.

Depressive Disorders

K&S Ch. 24

Question 43. E. In pyromania the patient sets fires repeatedly because of the tension before the act and the relief after. There is also a fascination with fire and its various uses. If the patient is setting fires for gain such as money or to make a political statement, then it is not a case of pyromania. One can not make the diagnosis in the presence of conduct disorder, mania, or antisocial personality disorder. Pyromania is included in the impulse control disorders.

Disruptive, Impulse Control, Conduct Disorders, and ADHD

K&S Ch. 25

Question 44. A. Apoptosis is the phenomenon of programmed cell death. Apoptosis can be triggered by exposure to antigens, exposure to corticosteroids, withdrawal of growth factors and cytokines. NMDA receptor channels, when opened, lead to calcium influx into neuronal cells and this triggers the apoptotic cascade known as hyperexcitability and excitotoxicity that leads to neuronal compromise and demise. This mechanism of cellular death is not well understood, but is believed to be implicit in the mediation of such neurologic conditions as epilepsy, stroke and neurodegenerative diseases like Alzheimer’s dementia and amyotrophic lateral sclerosis.

Neurology

B&D Ch. 41&60

Question 45. E. This is a case of intermittent explosive disorder. The patient has a history of violent outbursts that are out of proportion to the severity of the situation in which they occur. He has no other signs or symptoms that would suggest another axis I or II diagnosis. He has no medical history that would suggest that he could have a seizure disorder or that would otherwise explain his behavior.

Disruptive, Impulse Control, Conduct Disorders, and ADHD

K&S Ch. 25

Question 46. B. The treatment of choice for acute benzodiazepine overdose is flumazenil (Romazicon). Flumazenil is administered intravenously and has a short half-life of 7–15 minutes. The initial dosage in suspected benzodiazepine overdose is 0.2 mg IV over 30 seconds. An additional 0.3 mg can be given for more efficacy. Further doses of 0.5 mg can be given to a maximum total of 3 mg. The most common serious side effect of flumazenil administration is the onset of seizures, particularly in those patients who are dependent on benzodiazepines, or in those who have ingested large quantities of benzodiazepines.

Naloxone (Narcan) is administered in cases of acute opioid intoxication or overdose. Dimercaprol (British Anti-Lewisite) is a chelating agent that is administered in cases of acute lead poisoning and lead encephalopathy. Atropine is a potent anticholinergic agent that is administered in cases of organophosphate or herbicide poisoning. Epinephrine injection is administered in cases of allergic anaphylactic shock.

Psychopharmacology

K&S Ch. 36

Question 47. D. All of the answer choices in this question are potential considerations for the differential diagnosis of intermittent explosive disorder except obsessive–compulsive disorder (OCD). OCD patients do not have a particular likelihood to become intermittently violent and destructive. Other things to include in the differential would be personality change from a medical condition, oppositional defiant disorder, antisocial personality disorder, mania, malingering, and schizophrenia.

Disruptive, Impulse Control, Conduct Disorders, and ADHD

K&S Ch. 25

Question 48. B. The clinical triad of infantile spasms, hypsarrhythmia and psychomotor developmental arrest is known as West’s syndrome. The incidence is about 1 in 5000 live births. The onset of symptoms is generally before 1 year of age. Spasms occur intermittently and involve rapid flexor/extensor movements of the body. Developmental arrest occurs prior to or at the onset of spasms. Electroencephalography reveals a characteristic interictal hypsarrhythmia pattern, which is a pattern with predominant posterior diffuse slow and sharp waves and spikes. Treatment of choice is adrenocorticotropic hormone, prednisone, or prednisolone. Prognosis is extremely poor. Only 5% of affected children achieve normal development.

The 3-per-second spike and wave pattern is characteristic of absence seizures. PLEDS are characteristic of herpes simplex virus encephalitis. Triphasic sharp waves are characteristic of Creutzfeldt–Jakob disease. Burst-suppression pattern is indicative of severe diffuse brain damage, such as in anoxic brain injury, severe drug overdose and head injury.

Neurology

B&D Ch. 67

Question 49. C. Heat intolerance is a symptom of hyperthyroidism, not AIDS. The other symptoms in this question, such as progressive dementia, personality changes, depression, and loss of libido are all worth considering in a patient with AIDS. More than 60% of AIDS patients have neuro-psychiatric symptoms. They also show impaired memory, decreased concentration, and may have seizures.

Neurocognitive Disorders

K&S Ch. 11

Question 50. A. Papez’ circuit connects the hippocampus with the thalamus, hypothalamus and cortex. The pathway includes all of the answer choices noted, plus the mammillothalamic tract and anterior nucleus of the thalamus. The amygdala is not considered part of Papez’ circuit.

Neurology

B&D Ch. 6

Question 51. E. Oh joy! Mahler’s stages of separation–individuation are back again! This question focuses on Mahler’s last stage of object constancy, which lasts from 2 to 5 years. In this stage the child understands the permanence of other people, even when they are not present. For more information on Mahler’s stages, see Question 21.

Human Development

K&S Ch. 2

Question 52. E. Benign familial neonatal convulsions is a rare, autosomal dominant hereditary disorder resulting from a defect in voltage-gated potassium channels. Generalized tonic–clonic seizures occur after about the third day of life and disappear spontaneously in most cases in a few weeks to months.

Lambert–Eaton myasthenic syndrome is a paraneoplastic autoimmune disorder affecting P/Q-type voltage gated calcium channels at the motor neuron terminal. Malignant hyperthermia, hypokalemic periodic paralysis and familial hemiplegic migraine are all genetic disorders involving gene mutations that result in abnormal voltage-gated calcium channels.

Neurology

B&D Ch. 64

Question 53. C. The patient in this question has paranoid personality disorder. Symptoms not listed in this question include reluctance to confide in others for fear that the information will be used maliciously against one, perceiving attacks on one’s character that others do not see, and having recurrent suspicions regarding fidelity of sexual partners. The patient in the question does not present with the usual grouping of positive and negative symptoms used to diagnose schizophrenia. Although there is a great deal of suspiciousness present she does not present with frank psychosis. There is an absence of bizarre beliefs on a number of subjects, magical thinking, and excessive social anxiety as would be expected in schizotypal personality disorder. She does not exhibit ego-syntonic social isolation, as would be expected in schizoid personality disorder. There is no suggestion of memory loss as would be noted in dementia.

Personality Disorders

K&S Ch. 27

Question 54. A. Phenylketonuria (PKU) is an autosomal recessive heritable inborn error of metabolism resulting from a deficiency in phenylalanine hydroxylase that can cause mental retardation. The prevalence of the disorder is about 1 in 10 000 to 20 000 live births. Phenylalanine levels increase dramatically because it cannot be converted to tyrosine due to the missing enzyme. The defect localizes to chromosome 12. Classic clinical features include microcephaly, a characteristic ‘mousy’ odor, infantile spasms, and light hair and skin pigmentation. Sensorineural deafness is not a feature of PKU. Diagnosis can be established by measurement of blood levels of phenylalanine which are elevated. The treatment of choice is a phenylalanine free diet, which must begin during gestation in order to prevent mental retardation.

Neurology

B&D Ch. 62

Question 55. C. The nigrostriatal tract projects from the substantia nigra to the corpus striatum. When the D2 receptors in this tract are blocked, parkinsonian side effects emerge. This tract degenerates in Parkinson’s disease. Choices A, B, and C are all tracts involved in some way with antipsychotic medications. The antipsychotic medications effect both positive and negative symptoms of schizophrenia through the mesolimbic–mesocortical tract. They also lead to increased prolactin, amenorrhea, and galactorrhea through the tuberoinfundibular tract. The caudate neurons have many D2 receptors as well, and regulate motor activity. With blockade of the caudate D2 receptors bradykinesia develops. With their over stimulation tics and extraneous motor movements develop.

Neurocognitive Disorders

K&S Ch. 3

Question 56. D. The photo depicts a classic Arnold–Chiari type I malformation. The Chiari I malformation presents as a descent of the cerebellar tonsils below the level of the foramen magnum with or without forward displacement of the medulla. This manifestation is believed to be caused by a low intracranial pressure state. The defect is often accompanied by syringomyelia, syringobulbia, and hydrocephalus. Common clinical features include headache, cranial neuropathies, and visual disturbances. Some cases present with motor and sensory complaints, particularly myelopathy with a “shawl” distribution pattern of sensory deficit over the shoulders due to the syrinx. Surgical decompression of the brainstem may be needed if the patient is highly symptomatic. Chiari type II malformation has similar features as type I, with caudal displacement of the medulla and fourth ventricle and the addition of a lumbar myelomeningocele.

image

Lissencephaly, also known as agyria, is a disorder of early neuroblast migration. It results from the developmental failure of the gyri of the cerebral cortex. The cortex remains smooth and lacks the convolutions that are typical of normal neuronal migration and development. Schizencephaly is a disorder associated with defective genetic expression of the EMX2 gene. It results in clefts in the cerebral hemispheres. There are two types of schizencephaly: closed lip (the edges of the cleft are closed) and open lip (the edges of the cleft are open). Schizencephaly can be symmetrical or asymmetrical. Dandy–Walker syndrome presents as a cystic enlargement or pouching of the fourth ventricle. In addition, the posterior portion of the cerebellar vermis is hypoplastic or aplastic. Noncommunicating hydrocephalus invariably results from this anomaly. Mental retardation and spastic diplegia are frequent clinical manifestations.

Neurology

B&D Ch. 60

Question 57. D. Neuroleptic malignant syndrome is a life-threatening condition resulting from the use of antipsychotic medications. Its symptoms are muscular rigidity, dystonia, akinesia, obtundation, or agitation. It also involves autonomic instability such as fever, sweating, unstable blood pressure, or unstable heart rate. Patients are given supportive medical treatment and medications such as dantrolene and bromocriptine may be used. Mortality is around 10–20%. The syndrome is a result of dopamine blockade, and some postulate it may be the result of a precipitous withdrawal of dopamine receptor stimulation. The other answer choices would not present with the grouping of symptoms listed, and the fact that the patient has schizophrenia and is on a psychiatric inpatient unit should point the test-taker in the direction of a side effect to antipsychotics.

Psychopharmacology

K&S Ch. 36

Question 58. C. About 15% of ischemic strokes are caused by extracranial internal carotid artery (ICA) stenosis. Endarterectomy, surgical removal of the atherosclerotic plaque, has been deemed to be indicated and useful in symptomatic ICA stenosis of 70–99%. Symptomatic stenosis implies the prior occurrence of ipsilateral ischemic events such as transient ischemic attacks, amaurosis fugax, or completed nondisabling carotid territory stroke within the past 6 months. Asymptomatic ICA stenosis is a more difficult and controversial issue with respect to its optimal surgical management. European stroke trials and many stroke experts feel that asymptomatic ICA stenosis of 80% or greater presents a high enough risk to warrant endarterectomy only if carried out by a surgeon with morbidity statistics indicating less than a 3% complication rate.

Neurology

B&D Ch. 51

Question 59. A. The patient in this question has schizoid personality disorder. He has a pervasive pattern of detachment from social relationships and a restricted range of emotional expression. Other symptoms include taking little to no pleasure in activities, lacking close friends or confidants, and showing emotional coldness or detachment. Schizotypal patients have odd behaviors and beliefs such as magical thinking. Narcissistic patients are grandiose, need admiration, and lack empathy. This patient is socially isolated but has none of the other criteria that one would expect to find with a major depressive disorder or dysthymic disorder, and as such they are not the correct answers.

Personality Disorders

K&S Ch. 27

Question 60. D. Campylobacter jejuni is the most common bacterial infection that precedes the onset of Guillain-Barré syndrome (acute inflammatory demyelinating polyneuropathy; AIDP). It accounts for about 20–40% of all cases of AIDP. C. jejuni causes an acute enteric and systemic illness and the onset of AIDP occurs about two to three weeks after this initial diarrheal condition. AIDP tends to follow about 1 in 1000 to 2000 cases of known C. jejuni infections. C. jejuni infection can be diagnosed by stool or blood cultures and is treated with a week-long course of oral erythromycin 250 mg four times daily.

AIDP is preceded by an upper respiratory or gastrointestinal infection, a surgical intervention, or an immunization about 1 to 4 weeks prior to the illness onset in about two-thirds of cases. C. jejuni infection has been linked to a worse prognosis because it tends to be associated with the more severe axonal form of AIDP.

Neurology

B&D Chs 53&76

Question 61. C. The correlation coefficient is a measurement of the direction and strength of the relationship between two variables. The Pearson correlation coefficient is on a scale from −1 to +1. A positive correlation means that one variable moves the other in the same direction. A negative value means that one moves the other in the opposite direction. A correlation close to −1 or +1 shows a strong relationship. A correlation close to 0 shows a weak relationship. Correlation coefficients indicate the degree of relationship only; they say nothing about cause and effect.

Statistics

K&S Ch. 4

Question 62. D. Brain metastases are the most common form of neurologic complication from systemic cancer and they account for about 20–40% of all central nervous system tumors. Lung cancer is the most common cause of metastases to the brain and accounts for about two-thirds of all such metastases. In particular, non-small cell lung carcinoma accounts for about two-thirds of all metastases to the brain originating from the lung. Breast cancer leads to about 15–20% of patients with brain metastases. Melanoma, gastrointestinal carcinoma and renal cell carcinoma each account for about 5–10% of metastatic brain tumors.

Neurology

B&D Ch. 52

Question 63. B. This question gives a description of avoidant personality disorder. These patients avoid interpersonal contact for fear of criticism. They are unwilling to get involved with others without assurance of being liked. They show restraint in interpersonal relationships for fear of being shamed or ridiculed. They are inhibited interpersonally because of fears of inadequacy. They see themselves as inferior to others. Of the other choices, the only one that comes close is schizoid personality disorder, but in schizoid personality disorder the person doesn’t care that he is isolated, and he has a blunted affect. Passive–aggressive personality disorder is only in the DSM-IV-TR as a subject for further research. Obsessive–compulsive personality disorder (OCPD) presents as a pervasive pattern of preoccupation with orderliness, perfectionism, and control. This preoccupation comes at the expense of openness, efficiency, and flexibility. The OCPD patient’s perfectionism interferes with task completion. These patients are inflexible regarding moral and ethical issues. They devote time to work at the expense of leisure activities. They are reluctant to delegate tasks to others. They are characteristically rigid and stubborn. OCPD patients often can not discard old or worn-out objects even when they have no value.

Personality Disorders

K&S Ch. 27

Question 64. A. A post-lumbar puncture (LP) headache is a frequent adverse event resulting from diagnostic or therapeutic LP. The headache is considered to be due to low cerebrospinal fluid (CSF) pressure because the needle bevel (usually a Quincke needle) leaves an open dural tear from shearing as it is pulled out of the patient. This dural tear results in a chronic CSF leak from the puncture site and an intractable headache that is noted particularly when the patient is upright. The headache is generally relieved when the patient lies recumbent. Loss of CSF results in a traction of the brain on sensory nerves and bridging veins, which causes pain.

The pain often resolves in several days if the patient lies recumbent and receives adequate hydration. A blood patch is often immediately curative of the post-LP headache and involves the gentle injection of 10 to 20 mL of the patient’s own blood into the epidural space at the same site of the LP. Bed-rest for 2 weeks is far too long a period to wait before undertaking a blood patch. Acetaminophen and hydrocodone may be helpful, but are not curative therapy for post-LP headache. Sumatriptan is indicated for migraine only and not for post-LP headache.

Neurology

B&D Ch. 69

Question 65. E. Patients starting lithium should obtain baseline thyroid function tests, electrolytes, white blood cell count, renal function tests, and a baseline electrocardiogram. Why? Because lithium can cause renal damage, hypothyroidism, increased WBC count, and ECG changes (T-wave flattening or inversion). Low sodium can lead to toxic lithium levels. VDRL is a distracter which is unrelated to starting a patient on lithium.

Laboratory Tests in Psychiatry

K&S Ch. 7

Question 66. B. Compression of the lateral femoral cutaneous nerve as it passes beneath the inguinal ligament results in a painful sensory syndrome known as meralgia paresthetica. Predisposing factors include obesity, pregnancy and the wearing of pants that are tight at the waist or a heavy belt, such as those worn by workmen carrying heavy tools. Clinically there is pain and sensory loss (numbness) to the lateral thigh. Motor deficits are absent. Treatment is conservative and involves weight loss (if that is the cause), wearing looser clothing, or the use of tricyclic antidepressants or anticonvulsants to treat the neuropathic pain. Surgery is not generally needed in these cases. The other peripheral nerves listed as answer choices are simply distracters.

Neurology

B&D Ch. 30

Question 67. A. Your new friend Mahler is back again! If you didn’t get this right it’s time to memorize her stages! Mahler’s first stage is normal autism, lasting from birth to 2 months. In this stage the baby spends more time asleep than awake. For more information on Mahler’s stages, see Question 21.

Human Development

K&S Ch. 2

Question 68. B. Fibrillation potentials are the electromyographic (EMG) hallmarks of muscle denervation. Positive sharp waves are spontaneous activity emanating from groups of denervated muscle fibers. Fibrillation potentials, combined with the finding of positive sharp waves, and decreased recruitment in a segmental myotomal distribution are characteristic of a radiculopathy. Fasciculations are commonly characteristic of anterior horn cell diseases such as poliomyelitis and amyotrophic lateral sclerosis. They can also be benign and can be seen in normal healthy individuals. They represent spontaneous discharges of a single motor unit. High-frequency, short-duration potentials are seen when the EMG needle is very close to or embedded in the motor endplate. This is not a pathologic finding. Myotonia is the phenomenon of delayed muscle relaxation after needle insertion or contraction. It is the pattern noted in myotonic dystrophy and is characterized by a “dive-bomber” pattern heard on the speaker. Small, short motor unit potentials are noted in myopathies (polymyositis, muscular dystrophies). This pattern occurs from reduction in size of the muscle fibers.

Neurology

B&D Ch. 32

Question 69. A. The caudate nucleus neurons have many D2 receptors. They regulate motor activity by determining which motor acts get carried out. With blockade of the caudate D2 receptors, bradykinesia develops from excessive dampening of motor activity. With caudate D2 receptor over-stimulation, tics and extraneous motor movements develop.

Basic Neuroscience

K&S Ch. 3

Question 70. B. Cerebral venous thrombosis (CVT) generally occurs 1 day to 4 weeks postpartum, with a peak in incidence about 7–14 days after delivery. Clinical features include puerperal headache that worsens over days, seizures, neurologic deficits, and behavior or personality changes. The condition is believed to be due to the hypercoagulable state induced by pregnancy with likely decreased protein S activity and to a possible presence of circulating antiphospholipid antibodies during the puerperal period. Brain MRI with MRI venography is the imaging modality of choice to clinch the diagnosis. Heparin anticoagulation may be helpful. Thrombolysis by either intravenous infusion or invasive venographic approach has also proved to be useful treatment, especially in severe cases where prognosis is deemed to be poor.

Meningitis and meningoencephalitis are certainly in the differential diagnosis when considering CVT, but both would be expected to be accompanied by high fever and possibly obtundation. A migraine, even when complicated, is rarely accompanied by seizures. Pseudotumor cerebri does not cause seizures and usually presents with intermittent visual obscurations and papilledema rather than diplopia.

Neurology

B&D Ch. 81

Question 71. A. Patients must make decisions regarding advanced directives at a time when they are competent to do so. At the time of admission to the hospital they may or may not be competent. If the question of capacity arises they may or may not have capacity and therefore lack competence. When immediately prior to an operation, or in the office, they may or may not be competent. Competence is a legal decision, made by the courts, that a patient has sufficient ability to manage their own affairs. Capacity is a medical decision made by a psychiatrist that says whether at a given point in time the patient is thinking clearly enough to make certain medical decisions for himself. In order to make an advanced directive one would want the patient to have both capacity and be competent at the time of the decision.

Forensic Psychiatry

K&S Ch. 57

Question 72. A. Cytomegalovirus (CMV) is the most common congenital viral infection in newborns and is the result of either primary maternal infection or from viral reactivation in the mother. Most affected newborns are asymptomatic and most develop normally. Less than 10% of patients with CMV have complications such as jaundice, hepatosplenomegaly, microcephaly, chorioretinitis, ataxia, and seizures. The mortality rate is about 20–30% in symptomatic newborns. The other neonatal infections noted in this question occur less frequently than CMV.

Neurology

B&D Ch. 80

Question 73. D. Patients with dependent personality disorder have trouble making decisions without excessive amounts of advice from others. They need others to assume responsibility for most areas of their life. They do not express disagreement with others for fear of disapproval. They feel helpless and uncomfortable when alone. They urgently seek new relationships when a prior one ends, and can be unrealistically preoccupied with fears of being left alone to care for themselves. None of the other answer choices fit these parameters. Passive–aggressive personality disorder is in the DSM as a subject for further research.

Personality Disorders

K&S Ch. 27

Question 74. D. Inclusion body myositis is the most common myopathy in those over 50 years of age and it rarely occurs in those younger than 50 years of age. Men are more frequently affected than women. The disorder affects strength in the distal muscles of the arms and legs. Wrist and finger flexors and quadriceps muscles are preferentially weak. There are no muscle pains noted in the disorder. The disorder is generally chronic, progressive and poorly responsive to corticosteroid or immunosuppressive therapies. Muscle biopsy helps confirm the diagnosis in about 80% or more cases if done properly. It classically reveals endomysial inflammation, macrophage invasion of muscle, rimmed vacuoles, and characteristic inclusion bodies in the nuclei.

Facioscapulohumeral dystrophy (FSHD) is an autosomal dominant inherited disease of muscle with a prevalence of about 1 to 2 per 100 000. The genetic anomaly localizes to 4q35 in most cases. The phenomenon of anticipation is noted in FSHD, which implies that as successive generations acquire the disease, the onset of the condition occurs earlier and the disease becomes more severe. Clinical features include weakness of orofacial muscles, with inability to pucker or whistle. Shoulder muscles are weak and winging of the scapula can be noted when the arms are outstretched. Biceps and triceps are often weak, as are the hip flexors and quadriceps. DNA studies establish the diagnosis. Treatment is supportive as the condition is irreversible.

Oculopharyngeal muscular dystrophy (OPMD) is an autosomal dominant inherited disease of muscle. The disorder localizes to chromosome 14q11.2-13. The disease begins in the fifth or sixth decade most often and presents initially with eye muscle weakness and ptosis. Difficulty swallowing soon follows and swallowing may become impossible. Death can occur from starvation if nutritional support is not given. DNA testing proves the diagnosis. Treatment is supportive.

Dermatomyositis and polymyositis are discussed at length in other questions in this volume.

Neurology

B&D Ch. 79

Question 75. C. Euphoria is a behavioral effect of opioids, as are drowsiness, decreased sex drive, hypoactivity, and personality changes. Miosis is a physical effect of opioid use. Increased arousal is not an effect of opioid intoxication, drowsiness is. Diarrhea can come from withdrawal, but opioids themselves cause constipation. Bradycardia is also a physical effect of opioids, and as such tachycardia is incorrect.

Substance Abuse and Addictive Disorders

K&S Ch. 12

Question 76. A. Duchenne’s and Becker’s muscular dystrophies are X-linked inherited disorders of muscle. The gene locus is Xp21 on the short arm of the X chromosome. This abnormality results in a deficiency in dystrophin, a structural muscle membrane protein located in the subsarcolemmal region of muscle fibers. These two disorders are explained in greater detail as part of other questions in this volume. The other answer choices are all muscle proteins that work together with dystrophin to stabilize the muscle membrane, but they are not implicated in the pathophysiology of the two muscular dystrophies mentioned in this question.

Neurology

B&D Ch. 79

Question 77. E. Sleep apnea is a disorder in which there is a cessation of airflow in and out of the lungs during sleep. These stoppages of airflow must last for 10 seconds or more. In central sleep apnea, both respiratory effort and airflow stop. In obstructive sleep apnea, air stops flowing but respiratory effort increases. It is considered pathological if patients have five or more apneic episodes per hour or 30 or more episodes per night. Sleep apnea can lead to cardiovascular changes including arrhythmia and blood pressure changes. Long-standing sleep apnea can lead to pulmonary hypertension. The characteristic pattern of sleep apnea involves an older person who reports tiredness or inability to stay awake during the daytime. It can be associated with depression, irritability, and daytime sleepiness. Bed partners often report loud snoring. Patients may also awaken during the night as a result of the cessation of breathing. Patients suspected of sleep apnea should undergo sleep studies, and should be treated with a CPAP machine (continuous positive airway pressure). Losing weight helps many people, and for some surgery is the appropriate option to remove the obstruction in the airway. Now looking at the test question, the patient is not dropping into “sleep attacks” while in the middle of activities, as such narcolepsy is incorrect. There is no substantial evidence of mania given. The patient does not meet criteria for major depression. Behaviors associated with sleep terrors are not described. The snoring that drove her husband out of the bedroom should immediately point you in the direction of sleep apnea. The fact that she is overweight clinches the diagnosis.

Sleep Wake Disorders

K&S Ch. 24

Question 78. B. Tyrosine hydroxylase is the rate-limiting enzyme in the dopamine synthetic pathway. Dopamine synthesis occurs as follows: L-tyrosine is converted to L-dopa by tyrosine hydroxylase. Dopa decarboxylase converts L-dopa to dopamine. Once dopamine is extruded into the synaptic cleft, its termination of action is carried out by monoamine oxidase, catechol-O-methyltransferase, as well as reuptake into the presynaptic bouton where it is initially synthesized. Norepinephrine synthesis occurs when dopamine β-hydroxylase converts dopamine to norepinephrine. Norepinephrine in turn is converted to epinephrine by phenylethanolamine N-methyltransferase (PNMT).

Neurology

B&D Ch. 71

Question 79. C. The therapeutic range for lithium is 0.6–1.2 mEq/L. Toxic levels are 2 mEq/L or higher. Lethal levels are 4.0 mEq/L or higher.

Laboratory Tests in Psychiatry

K&S Ch. 7

Question 80. D. GABA-A is a complex receptor with multiple binding sites. GABA is found throughout the central and peripheral nervous systems and is the predominant inhibitory neurotransmitter in the brain. When the GABA receptor is agonized, there is a rapid influx of negatively charged chloride ions through the postsynaptic cellular membrane. This results in fast inhibitory post-synaptic potentials. The GABA-A receptor is believed to be responsible for the clinical effects of benzodiazepines, barbiturates and alcohol. Only sodium oxybate (gamma hydroxybutyrate; GHB; Xyrem), which is a “date-rape” drug that is FDA-approved for narcolepsy and cataplexy, and Lioresal, a potent antispasticity agent, act in the central nervous system by agonism of the GABA-B receptor. The other receptor types mentioned in this question are simply distracters.

Neurology

Basic Neuroscience

B&D Ch. 71

Question 81. D. It is the frontal lobes that determine how the brain acts on information. The relatively large size of the human frontal lobes is what distinguishes our brains from those of primates. It is in the frontal lobes that executive functioning takes place, and injury of the frontal lobes leads to impairment in motivation, attention, and sequencing of actions. A “frontal lobe syndrome” exists, and consists of slowed thinking, poor judgment, decreased curiosity, social withdrawal, and irritability. Patients with frontal lobe dysfunction may have normal IQ, as IQ has been found to be mostly a parietal lobe function.

Basic Neuroscience

K&S Ch. 3

Question 82. B. Ganser’s syndrome, considered to be a dissociative disorder, is the voluntary production of symptoms that involve giving approximate answers or talking past the point. This syndrome is often associated with other psychopathy such as conversion, perceptual disturbances, and dissociative symptoms like amnesia and fugue. Males and prisoners are most commonly affected. The major contributory factor is the presence of a severe personality disorder. Recovery is most often sudden and patients claim amnesia of the symptoms. It is believed to be a variant of malingering, with possible secondary gain.

Conversion disorder is a somatoform disorder characterized by the presence of one or more neurologic symptoms that are not explained by any known neurologic or medical disorder. Capgras’ syndrome is a specific type of systematized delusion in which the patient feels that a familiar person is mistakenly thought to be an unfamiliar imposter. Hypochondriasis is a somatoform disorder in which the patient misinterprets bodily symptoms and functions and becomes preoccupied with the fear of contracting or having a serious disease even after reassurance to the contrary is given by a physician. Folie-à-deux is a delusional disorder, now termed a shared psychotic disorder in DSM-IV-TR. It involves the transfer of delusions from a patient to another person who has a close relationship with the patient. The associated person’s delusion is similar in content to that of the patient.

Somatic Symptom Disorders

K&S Ch. 20

Question 83. D. Learned helplessness is a behavioral model for depression developed by Martin Seligman. He took dogs and gave them electric shocks from which they could not escape. Eventually they gave up and stopped trying to escape. In time this spread to other areas of functioning until they were always helpless and apathetic. This behavioral pattern has also been seen in humans with repeated setbacks or failures in their lives, as the question stem demonstrates. Industry is part of Erikson’s stages and is irrelevant to the question. Cognition is the process of obtaining, learning, and using intellectual knowledge. This has some relation to the test taking but is not the explanation for the child’s behavior. Sensory deprivation is removing a person or animal from external stimuli of any kind. Again, it is unrelated to the question. The epigenetic principle states that development occurs in sequential, clearly defined stages. This is clearly unrelated to the question.

Psychological Theory and Psychometric Testing

K&S Ch. 4

Question 84. A. Entacapone (Comtan) and tolcapone (Tasmar) are catechol-O-methyl-transferase (COMT) inhibitors. COMT inhibitors block peripheral degradation of peripheral levodopa and central degradation of L-dopa and dopamine, thereby increasing L-dopa and dopamine levels centrally. The COMT inhibitor is generally given concomitantly with each dose of carbidopa-levodopa (Sinemet) throughout the day to improve parkinsonian symptoms.

Selegiline (Eldepryl) is the classic monoamine oxidase type-B (MAO-B) inhibitor. It has some potentiating effects on dopamine and prevents MAO-B dependent dopamine degradation. Phenelzine (Nardil) and tranylcypromine (Parnate) are nonspecific MAO inhibitors that affect both MAO-A and MAO-B. There are reversible MAO-A inhibitors (moclobemide and befloxatone) that are only available outside the USA. These agents do not require the strict dietary restrictions on tyramine that the nonselective MAO inhibitors require in order to avoid a hypertensive crisis. There are no specific pharmacologic agents that act upon dopa decarboxylase or dopamine β-hydroxylase. These two answer choices are simply distracters.

Neurology

B&D Ch. 71

Question 85. D. The commonly used drug ibuprofen can drastically increase lithium levels. Many diuretics can increase lithium levels, as can ACE inhibitors and other non-steroidal anti-inflammatory drugs such as naproxen. Aspirin will not affect lithium levels. Lithium combined with anticonvulsants can increase lithium levels and worsen neurotoxic effects.

Laboratory Tests in Psychiatry

K&S Ch. 36

Question 86. C. Argyll Robertson pupils are a characteristic of late syphilis, particularly in either general paresis or tabes dorsalis. Argyll Robertson pupils are small, irregular pupils that constrict to accommodation, but not to light. Tabes dorsalis is the spinal form of syphilis and develops about 15 to 20 years after the initial infection. The clinical triad is that of sensory ataxia, lightning pains and urinary incontinence. Lower extremity deep tendon reflexes are absent. There is impaired proprioception with a positive Romberg’s sign. Ninety per cent of patients have pupillary abnormalities and about one-half have Argyll Robertson pupils. Another classic characteristic is the presence of Charcot’s (neuropathic) joints.

The pupillary abnormality seen in optic neuritis or multiple sclerosis is called the Marcus Gunn pupil and is revealed when a swinging light is moved away from the affected eye. Both eyes should constrict because of consensual innervation, but the affected pupil enlarges when the flashlight is moved away to the unaffected eye. Lyme disease and bubonic plague do not present with characteristic pupillary abnormalities. Intracerebral aneurysm, particularly of the posterior communicating artery, presents with a complete third nerve palsy that involves the pupil which is fixed and slightly dilated.

Neurology

B&D Ch. 53

Question 87. C. The symptoms given in this question are most likely the result of a serotonin withdrawal syndrome, which is possible with most serotonin-selective reuptake inhibitors (SSRIs), but which occurs particularly with paroxetine due to its shorter half-life. Symptoms include nausea, insomnia, muscle aches, anxiety and dizziness. The way to prevent this uncomfortable situation is to taper the drug off slowly. Patients on paroxetine should be warned against abruptly stopping the drug.

Psychopharmacology

K&S Ch. 36

Question 88. B. Todd’s paralysis is a brief period of transient hemiparesis or hemiplegia following a seizure. The symptoms usually dissipate within 48 hours and treatment is expectant and supportive. The weakness is generally contralateral to the side of the brain with the epileptic focus. The condition may also affect speech and vision, but again the deficits are temporary. Certain studies have pointed to Todd’s paralysis being the result of arteriovenous shunting that leads to transient cerebral ischemia following an ictal event. The other answer choices are distracters.

Neurology

B&D Ch. 67

Question 89. A. The 3-hydroxy benzodiazepines are directly metabolized by glucuronidation and have no active metabolites. The 3-hydroxy benzodiazepines include oxazepam, lorazepam, and temazepam. Know these three well, as they are often the subject of questions on standardized exams. Some of the longest half lives are found with the 2-keto benzodiazepines (chlordiazepoxide, diazepam, prazepam) because they have multiple active metabolites that can keep working in the body from 30 hours to over 200 hours in patients who are slow metabolizers.

Psychopharmacology

K&S Ch. 36

Question 90. A. Tropical spastic paraparesis is a chronic progressive myelopathy associated with infection by human T-lymphotropic virus type-1 (HTLV-1). The condition affects men more often than women and the onset is usually after 30 years of age. Diagnosis is confirmed by cerebrospinal fluid polymerase chain reaction and detection of HTLV-1 antibodies. Clinical features include upper motor neuron weakness, bladder disturbance and variable sensory loss. By ten years out, 60–70% of patients cannot walk. Treatment can be undertaken with corticosteroids, interferon-α, or plasmapheresis, but has proven only minimally beneficial.

Neurology

B&D Ch. 74

Question 91. D. Lithium is often used to treat aggression in patients with schizophrenia, prisoners, those with conduct disorder, and the mentally retarded. It is less useful in aggression associated with head trauma and epilepsy. Other drugs used for aggression include anticonvulsants and antipsychotics.

Psychopharmacology

K&S Ch. 36

Question 92. C. The neurodegenerative disorders that are associated with expansion of genetic trinucleotide repeat sequences are: fragile X syndrome, myotonic dystrophy, Huntington’s disease, X-linked spinobulbar muscular atrophy, dentatorubral-pallidoluysian atrophy, spinocerebellar atrophies types 1, 2, 3, 6 and 7, and Friedrich’s ataxia. Multiple system atrophy is a Parkinson’s plus syndrome that is not associated with an expansion of trinucleotide repeat sequences.

Neurology

B&D Ch. 40

Question 93. B. The mesolimbic–mesocortical pathway projects from the ventral tegmental area (VTA) to many areas of the cortex and limbic system. This is the tract that is thought to mediate the antipsychotic effects of the antipsychotic medications. The nigrostriatal pathway is associated with parkinsonian effects of the antipsychotics. The caudate is associated with Tourette’s disorder and tics (see Question 69). The tuberoinfundibular pathway is associated with prolactin increase and lactation from antipsychotics.

Basic Neuroscience

K&S Ch. 3

Question 94. A. The “battle sign” is a hematoma overlying the mastoid that results from a basilar skull fracture extending into the mastoid portion of the temporal bone. The lesion is not usually visible until 2 to 3 days after the trauma. Frontal lobe damage would be expected to yield classic frontal lobe signs on examination, such as a Meyerson’s sign, rooting reflex, snout reflex, palmomental reflex, and grasp reflex. These signal extensive damage to the frontal lobe (or lobes if the sign is bilateral). Increased intracranial pressure presents with obtundation of level of consciousness, papilledema, and signs of brainstem compromise. Hypocalcemia that is chronic may result in the clinical observation of a Chvostek’s sign. The sign is positive when the cheek is tapped with the examiner’s finger and the corner of the mouth involuntarily contracts. Impending cerebral herniation usually presents with all of the signs of increased intracranial pressure as noted above. Systemic hypertension and respiratory compromise are also often noted.

Neurology

B&D Chs 5&56

Question 95. B. Fluoxetine and phenelzine should not be combined because one is a serotonin-selective reuptake inhibitor (SSRI) and the other is a monoamine oxidase inhibitor (MAOI). SSRIs should not be combined with MAOIs because of the possibility of causing a fatal serotonin syndrome. Using an SSRI with selegiline, which only inhibits MAO-B, is tolerated by some patients. But the general rule to remember is: do not mix SSRIs and MAOIs. If you give one followed by the other there must be a washout period in between. It would be a good idea for the well-prepared test taker to know the symptoms of a serotonin syndrome and how to distinguish it from neuroleptic malignant syndrome. Both conditions are covered elsewhere in this volume.

Psychopharmacology

K&S Ch. 36

Question 96. D. Atomoxatine (Strattera) is FDA-approved for symptoms of attention deficit–hyperactivity disorder (ADHD) in both adults and children. Its mechanism of action is by norepinephrine reuptake inhibition. It is the most useful choice when trying to treat both tics and ADHD, as it does not worsen the tic condition and may in fact help the tic symptoms. Bupropion (Wellbutrin) is not specifically FDA-approved for ADHD and its use in the disorder has not been shown to be consistently beneficial. The amphetamine-like stimulant medications methylphenidate (Ritalin, Metadate, Concerta) and dextroamphetamine (Dexedrine), although approved, well-studied and efficacious in ADHD, have been known to worsen underlying tics in concomitant Tourette’s syndrome. Clonidine (Catapres) is an alpha-2 adrenergic agonist that has little place in ADHD, although it may certainly help alleviate tics in Tourette’s syndrome.

Neurology

Psychopharmacology

B&D Ch. 61

Question 97. D. More statistics! Regression analysis is a method of using data to predict the value of one variable in relation to another. The other distracters are explained elsewhere in this volume.

Statistics

K&S Ch. 4

Question 98. C. Lioresal (Baclofen) is one of the most potent of the oral muscle relaxing agents and treats spasticity highly effectively. Its full mechanism of action is not well-understood, but it is believed to have predominant effect as a GABA-B agonist. It can cause muscular weakness and difficulty with weight-bearing because of its potency. Other first-line agents to treat symptomatic spasticity include gabapentin, diazepam, clonidine, tizanidine, and dantrolene. Second-line agents include intrathecal Lioresal, Marinol, chlorpromazine, cyproheptadine, phenytoin, and phenobarbital.

Neurology

B&D Ch. 48

Question 99. A. Just in case Question 61 wasn’t enough, here it is again. Correlation coefficient is a measurement of the direction and strength of the relationship between two variables. The Pearson correlation coefficient is on a scale from −1 to +1. A positive correlation means that one variable moves the other in the same direction. A negative value means that one moves the other in the opposite direction.

Statistics

K&S Ch. 4

Question 100. C. Triphasic waves on electroencephalogram are characteristic of hepatic or metabolic encephalopathy. Of course, hepatic encephalopathy is often accompanied by asterixis, particularly when the condition is severe. Asterixis is a sudden loss of postural tone and manifests as a flapping tremor of the hands. It is exhibited when the arms are fully extended. Herpetic skin vesicles would of course be expected with herpes simplex virus infections. Penile chancre can be demonstrated in cases of syphilis. Dupuytren’s contractures are a form of benign progressive fibroproliferative disease of the palmar fasciae of unknown etiology. The condition is seen more often in men than women and is associated with alcoholism, hand trauma and diabetes. Pulmonary rales would be an expected sign in congestive heart failure.

Neurology

B&D Ch. 56

Question 101. C. Experiments such as the one in this question have been used to give behavioral models for depression and stress. In this situation it is the unpredictability of the insults and the animal’s inability to control them that leads to a state of chronic stress. Animals put under chronic stress become restless, tense, irritable, or very inhibited. The concept of unpredictable stress is important to understand in conjunction with the theory of learned helplessness proposed by Martin Seligman. In learned helplessness experiments, animals are exposed to electric shocks from which they can not escape. They eventually become apathetic and make no attempts to escape. This can be generalized to human behavior as seen in a child who consistently fails in school and then gives up trying and becomes depressed. Learned helplessness is a proposed animal model for human depression. The other answer choices are unrelated, or are simply ridiculous.

Psychological Theory and Psychometric Testing

K&S Ch. 4

Question 102. D. Common side effects of valproic acid include weight gain, tremor, thinning of the hair, and ankle swelling. Other noted adverse effects are gastrointestinal distress, sedation, pancreatitis, bone marrow suppression (pancytopenia), and hepatotoxicity. The agent is teratogenic and the most worrisome fetotoxic effect is the neural tube defect (spina bifida) which is dangerous during neurogenesis in the first trimester of gestation.

Psychopharmacology

B&D Ch. 67

Question 103. E. It is the doctor’s ethical obligation to treat the patient in question regardless of ability to pay. It is unethical to allow a suicidal patient to leave the hospital due to a dispute with the insurance company. Should the patient go and kill himself, the doctor, not the insurance company, is liable. After the patient is treated and released, the issue can be taken up more aggressively with the insurance company. As a physician, your first and most important obligation is to the patient, not to the insurance company or to your wallet.

Ethics

K&S Ch. 58

Question 104. B. Clonidine (Catapres) is a presynaptic α2-receptor agonist. It is FDA-approved as an antihypertensive agent. It acts by reducing the amount of norepinephrine that is released from the synaptic bouton. This effect decreases sympathetic tone and bodily arousal and activation. The agent diminishes the autonomic symptoms associated with opioid withdrawal, such as tachycardia, hypertension, sweating, and lacrimation. Atomoxetine (Strattera) FDA-approved for ADHD in children and adults is a norepinephrine reuptake inhibitor. The neuroleptic medications, both conventional and atypical, can cause α1 adrenergic antagonism and thereby cause orthostatic hypotension. Dopamine type 2 antagonism is the putative antipsychotic mechanism of all of the neuroleptic agents, both conventional and atypical. Serotonin antagonism is what makes an atypical antipsychotic atypical. It is in fact the ratio of D2 to 5-HT2 blockade that reduces the extrapyramidal side effects of the atypical neuroleptics.

Psychopharmacology

K&S Ch. 36

Question 105. D. Heroin can be detected on a urine toxicology screen for 36–72 hours. Alcohol can be detected for up to 12 hours. Amphetamines can be detected for up to 48 hours. Cannabis can be detected for up to 4 weeks. PCP can be detected for up to 8 days.

Substance Abuse and Addictive Disorders

K&S Ch. 7

Question 106. E. The Klüver–Bucy syndrome results from bilateral destruction of the amygdaloid bodies and the inferior temporal cortex. Clinical features include hypersexuality, placidity and hyperorality. One of the causes of the syndrome is Pick’s disease (frontotemporal dementia). Other causes include stroke and Alzheimer’s dementia. Other associated features include visual agnosia (psychic blindness), hyperphagia and prosopagnosia (the inability to recognize faces). The rest of the answer choices are simply distracters.

Neurocognitive Disorders

K&S Ch. 10

Question 107. E. Patients at high risk during electro-convulsive therapy (ECT) include the following: those with space occupying lesions in the CNS, those with increased intracranial pressure, those at risk for cerebral bleed, those who have had a recent myocardial infarction, and those with uncontrolled hypertension. There are no absolute contraindications for ECT, but patients who fall into any of the above mentioned categories should be screened carefully and decisions made on a case by case basis depending on risks, benefits, and ability to control risk factors.

Diagnostic and Treatment Procedures in Psychiatry

K&S Ch. 36

Question 108. B. Klinefelter’s syndrome results from the presence of an extra X chromosome (XXY triploidy). It is noted in about 1 in 700 men. Clinical features include small dysfunctional testes, mental retardation and pear-shaped stature. Testosterone replenishment may offset some of the stigmata of the condition. Turner’s syndrome is the absence of an X chromosome in a genetic female (X0). Characteristics include short stature and lack of pubertal sexual development. Other clinical features include a webbed neck, and heart and kidney anomalies. Trisomy 21 is of course classic Down’s syndrome which is explained in detail in another question in this volume. Deletion on the paternal chromosome 15 results in the Prader–Willi syndrome. The prevalence is about 1 in 12 000 to 15 000. Clinical features include profound mental retardation, hypogonadism, hypotonia, behavioral disinhibition, rapid and excessive weight gain, and facial dysmorphism. Trisomy 18 is described elsewhere in this volume.

Neurology

B&D Ch. 40

Question 109. C. Patient controlled analgesia has proven to be an extremely good way of treating pain. Patients who control their own dosing end up using less pain medication than those who have to ask for the medication and wait for the doctor to write an order. They also have far better pain control. Some patients, particularly cancer patients may need large and escalating doses of medication to control their pain. Under these circumstances, this should not be viewed as addiction, but as a necessary part of the treatment of their illness. Cancer patients have been shown to wean themselves off of the pain medication once the pain decreases. The use of pain control in a drug addict with a painful medical illness is as important as it is in a non-addict with the same illness. They may need higher doses to control their pain, but the doctor has as much of an obligation to manage their pain as they do to manage the pain of the non-addict. The other answer choices do not address the issue of using as little medication as necessary while obtaining the best pain control. This is why patient-controlled analgesia is the best choice.

Somatic Symptom Disorders

K&S Ch. 28

Question 110. B. This vignette points to a diagnosis of intermittent explosive disorder. Intermittent explosive disorder manifests as discrete episodes of failure to resist aggressive impulses that lead to extreme physical aggression directed towards people and/or property. The degree of aggression is completely out of proportion to any particular psychosocial stressor that may trigger such an episode.

Episodes are unpredictable and often arise without cause or particular trigger and remit as spontaneously as they begin. There are no signs or symptoms of aggressivity noted in between these discrete episodes. The disorder is more common in men than in women. Predisposing psychosocial factors include an underprivileged or tempestuous childhood, childhood abuse, and early frustration and deprivation. Biological predisposing factors are believed to be decreased cerebral serotonergic transmission, low cerebrospinal fluid (CSF) levels of 5-hydroxyindoleacetic acid (5-HIAA) and high CSF levels of testosterone in men. There is strong comorbidity with fire setting, substance use and the eating disorders.

The personality disorders such as borderline and antisocial, are distinguished from intermittent explosive disorder by a pervasive pattern of maladaptive behavior that would be expected to occur in between episodes and affect the patient’s life adversely in more areas of functioning. Aggressive patients with bipolar mania would be expected to present with evidence of manic symptoms, including elevated/irritable mood, increased energy, rapid pressured speech, sleeplessness, racing thoughts, distractability, increased goal-directed behavior, and perhaps even psychosis. Temporal-lobe seizures are a remote possibility and can certainly result in aggression, most often interictally, but there is no evidence of this presented in this question.

Treatment of intermittent explosive disorder can be undertaken with mood stabilizers such as lithium, carbamazepine, divalproex sodium, and gabapentin. Selective serotonin reuptake inhibitors and tricyclic antidepressants can also be effective in reducing aggression.

Disruptive, Impulse Control, Conduct Disorders, and ADHD

K&S Ch. 25

Question 111. B. Catalepsy is an immobile position that is constantly maintained. Cataplexy is temporary loss of muscle tone precipitated by an emotional state. Psychomotor retardation is decreased motor and cognitive activity often seen with depression. Catatonia is markedly slowed motor activity to the point of immobility and unawareness of surroundings. Stereotypy is a repetitive fixed pattern of movement or speech.

Psychotic Disorders

K&S Ch. 8

Question 112. D. Bupropion (Wellbutrin) is the only antidepressant agent that is believed to be a dopamine and norepinephrine reuptake inhibitor. It is FDA-approved for both depression and smoking cessation. It has a unique side effect profile and is unlikely to cause the sexual dysfunction or weight gain noted with the serotonergic antidepressants. Bupropion is a noncompetitive inhibitor of nicotinic cholinergic receptors and in this way can reduce tobacco cravings in smokers. Bupropion is less likely to precipitate a bipolar manic episode than the tricyclic antidepressants. It is also less likely to induce rapid cycling in bipolar patients than other antidepressants.

Tiagabine (Gabitril) is a selective GABA reuptake inhibitor. It is FDA-approved for adjunctive therapy in partial complex seizures in adolescents and adults. Duloxetine (Cymbalta) is a serotonin and norepinephrine reuptake inhibitor. It is approved in both major depressive disorder and diabetic neuropathic pain. It works on pain modulation via the brainstem and spinal cord efferent noradrenergic tracts originating in the locus ceruleus. Venlafaxine XR is also a serotonin and norepinephrine reuptake inhibitor and is FDA-approved in major depressive disorder, generalized anxiety disorder, social anxiety disorder, and most recently obtained an indication in panic disorder. Atomoxetine (Strattera) is a norepinephrine reuptake inhibitor that is FDA-approved for the treatment of attention deficit–hyperactivity disorder in both children and adults.

Psychopharmacology

K&S Ch. 36

Question 113. D. The description in this question is of partial isolation monkeys. Those totally isolated from other monkeys were very fearful, unable to copulate, and unable to raise young. Mother-only raised monkeys failed to copulate, didn’t leave the mother to explore, and were scared of their peers. The peer-only raised monkeys were easily frightened, timid, had little playfulness, and grasped other monkeys in a clinging manner. In monkeys separated from their mothers, there was an initial protest stage followed by despair. Many of these behavior patterns can be correlated with human behaviors that are seen in our patients.

Psychological Theory and Psychometric Testing

K&S Ch. 4

Question 114. A. Sildenafil, vardenafil and tadalafil are nonselective phosphodiesterase 5 (PDE5) inhibitors. They also have some agonistic effects on nitric oxide (NO). PDE5 blockade causes arterial smooth muscle dilatation and facilitates cavernosal blood filling which potentiates penile erection. These agents cannot be used with nitrates because the combined effect of NO agonism with nitrates can cause significant vasodilatation and precipitous lowering of the blood pressure than can result in diminished cardiac perfusion and myocardial infarction. The other answer choices are simply distracters.

Psychopharmacology

K&S Ch. 36

Question 115. B. Amoxapine is one of the tricyclic antidepressants. What sets it apart from the others is that one of its metabolites has dopamine blocking activity. Because of this amoxapine has the potential to cause parkinsonian symptoms, akathisia, and even neuroleptic malignant syndrome. None of the other drugs listed has dopamine blocking activity.

Psychopharmacology

K&S Ch. 36

Question 116. C. The relative risk of an illness is the ratio of the incidence of the condition in those with risk factors to the incidence of the condition in those without risk factors. Attributable risk refers to the absolute incidence of the illness in patients exposed to the condition that can be attributed to the exposure. The other answer choices are nonsense distracters and are not true biostatistical terms.

Statistics

K&S Ch. 4

Question 117. C. Cingulotomy is a surgical treatment for obsessive–compulsive disorder. It is successful in treating about 30% of otherwise treatment resistant patients. Some patients who fail medication, and then subsequently fail surgery, will respond to medication after surgery. Complications of cingulotomy include seizures, which are then managed with anticonvulsants. The other disorders listed in this question do not have surgical treatments.

Diagnostic and Treatment Procedures in Psychiatry

K&S Ch. 16

Question 118. E. Dissociative identity disorder (DID) also known as multiple personality disorder, is a chronic dissociative disorder. The origins of the disorder are believed to stem from early childhood trauma, most often sexual or physical. The hallmark of the disorder is the presence of two or more distinct identities or personality states that recurrently take over the person’s behavior. There is also a presence of dissociative amnesia, with a noted inability to recall important personal information that is too extensive to be explained solely by forgetfulness.

The true cause of DID is unknown. Some research points to a possible connection between DID and epilepsy, with some patients having abnormal electroencephalograms. The absence of external supports, particularly from parents, siblings, relatives and significant others, seems to play a pivotal role in the genesis of the disorder. The patient’s lack of stress coping mechanisms is also a likely contributory factor. The differential diagnosis includes borderline personality disorder, rapidly-cycling bipolar disorder, and schizophrenia. The disorder can start at almost any age and an early age of onset is predictive of a worse prognosis.

Treatment is focused on insight-oriented psychotherapy. Hypnotherapy may also be helpful. Antipsychotic medications are often unhelpful. Antidepressant and anxiolytic medications can be useful in addition to psychotherapy. Anticonvulsant mood stabilizers have shown some efficacy in certain studies. Viral exposure or infection has nothing to do with the etiology of DID.

Dissociative Disorders

K&S Ch. 20

Question 119. D. The first-line therapy for the catatonic patient is intramuscular lorazepam. Many patients will respond to this treatment and will come out of their catatonic state. They can then be given subsequent treatment with antipsychotic drugs for the underlying psychotic disorder that is the most likely cause of the catatonia. Antidepressants and stimulants are not indicated for catatonia.

Psychotic Disorders

K&S Ch. 36

Question 120. C. Malingering is diagnosed in the presence of intentional production of symptoms that are exaggerated and either physical or psychological in nature. These symptoms are motivated by secondary gain and incentives to avoid responsibility or danger, or to obtain compensation or some other benefit that is material or monetary. Symptoms are vague, ill-defined, overdramatized and do not conform to known clinical conditions. Patients seek secondary gain most often in the form of drugs, money, or the avoidance of work or jail. History and examination typically do not reveal complaints from the patient. The patient is often uncooperative and refuses to accept a good prognosis or clean bill of health. Findings can be compatible with self-inflicted injuries. Medical records may have been tampered with or altered. Family members are usually unable to verify the consistency of symptoms.

Somatic Symptom Disorders

K&S Ch. 33

Question 121. D. Mutism refers to a patient who is voiceless without abnormalities in the structures which produce speech. Mutism is common in catatonic schizophrenia. It can be seen in conversion disorder. There is also a diagnosis known as selective mutism for children who consistently fail to speak in social situations despite speaking in other situations. Other causes of mutism include mental retardation, pervasive developmental disorder, and expressive language disorders. It can also be a component of buccofacial apraxia, locked-in syndrome, and a persistent vegetative state.

Psychotic Disorders

K&S Ch. 48

Question 122. B. Biofeedback is a therapy in which instruments are used to measure autonomic parameters in patients who are provided with “real-time” feedback from the instrumentation about their bodily physiologic processes. This feedback enables patients to control their own physiologic functions and alter them in positive ways to alleviate symptoms using operant conditioning techniques. Feedback is provided to the patient by measuring physiologic parameters such as heart rate, blood pressure, galvanic skin response and skin temperature. The measurement is translated into a visual or auditory output signal that patients can rely on to gauge their responses. Patients can alter the tone by using guided imagery, breathing techniques, cognitive techniques and other relaxation techniques. The modality is useful for anxiety disorders, migraine, and tension-type headache in particular.

Stimulus-response therapy is a nonsense distracter, as there is no such thing. Relaxation training is a form of behavior therapy that basically encompasses techniques such as meditation and yoga to help patients to dispel anxiety by tapping into their own physiologic parameters such as heart rate and breathing rate. Guided mental imagery also helps patients to enter a relaxed state of mind. Behavior therapy is the global term used to describe different therapeutic modalities that employ either operant or classical conditioning techniques to help patients overcome their fears, phobias and anxieties. Flooding, systematic desensitization, and aversion therapy are all examples of behavior therapy. Desensitization refers to the technique that helps patients gradually overcome their fears, phobias and anxieties, by graded exposure to the very stimulus that is the source of their fears. The patient is exposed to more and more anxiety-provoking stimuli, but relaxation training helps patients to cope with their maladaptive responses and eventually ideally respond to the stimulus without it evoking anxiety.

Diagnostic and Treatment Procedures in Psychiatry

K&S Ch. 35

Question 123. D. Lithium toxicity is a medical emergency and can result in permanent neuronal damage and death. Toxicity occurs at lithium levels above 2.5 mEq/L. Treatment includes discontinuation of lithium as well as vigorous hydration. If the level is above 4 mEq/L, or the patient shows serious signs of lithium toxicity (nephrotoxicity, convulsions, coma) the patient must have hemodialysis. Hemodialysis can be repeated every 6–10 hours until the level is no longer toxic and the patient’s symptoms remit. If the patient in question was not showing serious signs of toxicity, labs to assess the situation could be sent, neurological examination done, EKG obtained, gastric lavage performed, activated charcoal given, and vigorous hydration used. The patient could then be monitored and given time to clear the lithium.

Laboratory Tests in Psychiatry

Management in Psychiatry

K&S Ch. 36

Question 124. D. The most useful long-term treatment parameter for the noncompliant patient with schizophrenia who has a history of violence would be the use of an outpatient commitment program. Certain states have such laws in place, but others do not. Treating clinicians can petition the court to place refractory, potentially dangerous patients on this status. A judge mandates the patient’s cooperativeness and the patient is compelled to report for outpatient follow-up. If the patient is noncompliant, a psychiatrist can order that the patient be picked up against his wishes and brought in for psychiatric evaluation to an emergency room where the patient can be held for a period of time. Certain states use an alternative to outpatient commitment called conservatorship. This modality involves the court appointing a conservator, often a family member, to look after the patient and make decisions on the patient’s behalf, including placing the patient in involuntary hospitalization if it is deemed necessary.

Partial hospitalization is quite similar to inpatient hospitalization, except the patient sleeps at home. This modality is often a good transition between inpatient and outpatient services and patients can stay in such a program over extended periods of time if necessary. Patients have a case manager assigned to them in partial programs. The case manager helps coordinate and facilitate the patient’s care and helps make the patient’s transition easier to the less restrictive setting. Day treatment programs are somewhat less intense and less structured than partial hospital programs. Patients again sleep at home, but they participate in facility-based care five or more days a week. Case management and social work also play pivotal roles in the day treatment program setting.

Social skills training refers to a program that is dedicated to helping low-functioning patients, such as those with schizophrenia, to gain key skills that will enable them to function more independently in the community. The focus is on improving patients’ interactions with other people in their environment.

Public Policy

K&S Chs 35&59

Question 125. C. Monoamine oxidase inhibitors (MAOIs) have several serious drug–drug interactions than must be kept in mind. Meperidine (Demerol) can never be given with an MAOI, as this combination has led to death in several patients (common exam question!). MAOIs should never be used with anesthetics (no spinal anesthetics, no anesthetics containing epinephrine, lidocaine is OK). MAOIs should not be combined with asthma medication or over the counter drugs that contain dextromethorphan (cold and flu medications). They can not be given with sympathomimetics (epinephrine, amphetamines, cocaine). They can not be given with SSRIs or clomipramine as this will precipitate a serotonin syndrome. There are also many food restrictions with MAOIs which will, no doubt, be the subject of another question.

Psychopharmacology

K&S Ch. 36

Question 126. A. Privilege refers to the psychiatrist’s right to maintain a patient’s secrecy or confidentiality even in the face of a subpoena. This implies that the right of privilege belongs to the patient, not the psychiatrist and therefore the patient can wave the right. There are many exceptions to medical privilege, and many physicians are not aware that they do not legally enjoy the same privilege that exists between husband and wife, priest and parishioner, and a client and an attorney.

Confidentiality is the professional obligation of the physician to maintain secrecy regarding all information given to him by the patient. A psychiatrist may be asked to appear in court and testify by subpoena and thereby be forced to break a patient’s confidentiality. A patient may release the clinician from the obligation of confidentiality by signing a consent to release information. Each release pertains to a specific matter or piece of information and may need to be reobtained for subsequent disclosures.

Communication rights refers to the patient’s right to free and open communication with the outside world by either telephone or mail, while hospitalized. Private rights refers to the patient’s right to privacy. In a hospital setting, this applies to patients having private toileting and bathing space, secure storage space for personal effects, and adequate personal floor space per person. Patients also have the right to wear their own clothing and carry their own money if they desire to do so. Certain restrictions to this right may apply based on dangerousness to self or others. Clinical responsibility is not a forensic term per se, but it refers to the responsibility of the physician to the patient to provide the patient with the best care possible in any clinical setting, irrespective of the patient’s financial, racial, or personal status.

Forensic Psychiatry

K&S Ch. 57

Question 127. B. Patients being started on clozapine should have a baseline white blood cell (WBC) count with differential before treatment. A WBC count with differential is taken every week during treatment for the first 6 months, then every 2 weeks thereafter. When treatment is stopped WBC counts should be taken every week for 4 weeks. It is not a part of standard monitoring to take two WBC counts in one week. The US Food and Drug Administration recently approved monthly monitoring of WBC count after 12 months of therapy on clozapine.

Laboratory Tests in Psychiatry

K&S Ch. 7

Question 128. C. Patients over 65 years of age undergo sleep problems that affect both rapid eye movement (REM) sleep and non-rapid eye movement (NREM) sleep. There are more REM episodes noted. REM episodes are shorter in duration. There is less total REM sleep. In NREM sleep there is a decreased amplitude of delta waves. There is a lower percentage of stages three and four sleep. There is a higher percentage of stage one and two sleep. The elderly experience increased awakening after sleep onset.

Sleep Wake Disorders

K&S Ch. 55

Question 129. B. Clozapine has several drug–drug interactions that are noteworthy. Cimetidine, SSRIs, tricyclics, valproic acid, and erythromycin will all increase Clozaril levels. Phenytoin and carbamazepine will decrease clozapine levels. Clozapine should not be combined with any medication that can cause agranulocytosis (carbamazepine, propylthiouracil, sulfonamides, and captopril). CNS depressants (alcohol, benzodiazepines, and tricyclics) cause even more depression when combined with clozapine. The combination of lithium and Clozaril can increase neuroleptic malignant syndrome, seizures, confusion, and movement disorders. Other answer choices are distracters. The foods given should be avoided when taking MAOIs. Acetaminophen and aripiprazole have no interaction.

Psychopharmacology

K&S Ch. 36

Question 130. E. Cannabis is the most widely abused recreational drug among US high school students. Cannabis use has been demonstrated to lead to future cocaine abuse in adolescents. About 35% of high school seniors reported using cannabis. Alcohol is also a pervasive problem among high school teens, but only in about 10–20% of students surveyed. Over 85% of high school seniors have reported that they have tried alcohol at some time. About 15% of adolescents have reported using inhalants. Fewer than 2% of high school students report having used cocaine. About 9% of high school seniors have reported trying LSD at some time.

Substance Abuse and Addictive Disorders

K&S Ch. 51

Question 131. D. Aaron Beck is the originator of cognitive therapy. It is based on the theory that affect and behavior are determined by the way in which patients structure the world. Patients have assumptions, on which they base cognitions, which lead to affect and behavior. In depression specifically, Beck feels that there is a triad consisting of the following:

1. Depressed people see themselves as defective, inadequate, and worthless.

2. Depressed people experience the world as negative and self-defeating.

3. Depressed people have an expectation of continued hardship and failure.

It is this triad of distorted negative thoughts that Beck feels leads to depression. The goal of cognitive therapy is to help test the cognitions and develop more productive alternatives. Other answer choices given are distracters. Good enough mothering and transitional object are terms associated with the child development theory of Winnicott. Mania is not associated with Beck’s cognitive triad. Aggression toward the primary caregiver has nothing to do with Beck.

Psychotherapy

K&S Ch. 35

Question 132. D. Ramelteon (Rozerem) is a novel sleeping agent that was FDA-approved in 2005 for insomnia with sleep onset difficulties. It has a unique mechanism of action: it is a melatonin agonist. It works by stimulating melatonin type 1 and type 2 receptors in the suprachiasmatic nucleus of the hypothalamus. It has no addictive or abuse potential, because it is not a GABA-A agonist and has no activity at the benzodiazepine receptor whatsoever. It has no effects at histamine, acetylcholine, dopamine, serotonin, or norepinephrine receptors. The dosage is the same for all patients: one 8 mg tablet at bedtime. Recall that zolpidem, zaleplon and eszopiclone are all benzodiazepine receptor agonists and as such have sedative–hypnotic effects and potential for tolerance, withdrawal and abuse.

Psychopharmacology

K&S Ch. 36; also see www.rozerem.com

Question 133. E. Alogia is a lack of speech that results from a mental deficiency or dementia. Poverty of movement is called akinesia. Poverty of emotion is described using the term “flat affect.”

Diagnostic and Treatment Procedures in Psychiatry

K&S Ch. 8

Question 134. B. To meet criteria for the rapid cycling specifier in bipolar disorder, the patient must present with at least 4 mood episodes over the past 12 months. The mood episodes must meet criteria for a major depressive, manic, mixed, or hypomanic episode. Female patients are more likely than men to have rapid cycling bipolar disorder. There is no evidence to suggest that rapid cycling is a heritable phenomenon in bipolar disorder. It is therefore likely to be a result of external factors such as stress or medication.

Bipolar Disorders

K&S Ch. 15

Question 135. B. Random reinforcement is seen with the gambler. In random reinforcement the reward is only given a fraction of the time at random intervals. The money from a slot machine is won at random times. This keeps the gambler guessing and trying to anticipate when they will win. This is a very good way to maintain a behavior. Continuous reinforcement is when every action is rewarded, and is the best way to teach a new behavior. Primary reinforcers are independent of previous learning, for example, the need to eat is biological and not based on previous learning. Secondary reinforcers are based on previous learning, such as rewarding a child with a present when they do something well. The dexamethasone suppression test is an experimental measure associated with depression, and is unrelated to pathological gambling. Cerebellar dysfunction would lead to ataxia and gait disturbance, again, unrelated to pathological gambling.

Psychological Theory and Psychometric Testing

K&S Ch. 4

Question 136. B. Late-onset schizophrenia is noted more often in women than in men. The prognosis seems to be more favorable when the onset is late. There is a tendency to see more paranoia in these late-onset schizophrenic patients. Schizophrenia is considered late-onset when symptoms begin after 45 years of age. It is clinically identical to schizophrenia that has normal onset.

Psychotic Disorders

K&S Ch. 13

Question 137. B. One might think that this one was too easy, but it’s just the kind of question that could show up on an exam. A control group is a group in a study that does not receive treatment and is used as a standard of comparison.

Statistics

K&S Ch. 4

Question 138. E. Bowlby and Robertson identified three essential stages of separation response among children. The first stage is that of protest. The child protests the mother’s departure by crying, calling out and searching for her. The second stage is despair and pain. The child loses faith that the mother will return. The third stage is detachment and denial of affection to the mother figure upon her return. These phases are noted universally in children who go through separation by loss of parents to death, divorce, or going off to boarding school. Acceptance is not one of Bowlby’s stages of the separation response. It is the fifth and final stage of KüblerRoss’ stages of reaction to impending death.

Human Development

K&S Ch. 4

Question 139. C. Bupropion has been shown in studies to be efficacious for the treatment of attention–deficit/hyperactivity disorder (ADHD) in children and adults. It is also a very good antidepressant, which is its most common use. It has also been found very useful in patients who do not respond to a serotonin-selective reuptake inhibitor. When bupropion is added to SSRIs up to 70% of these cases improve, as both drugs together hit more neurotransmitter systems than either drug alone. The other drugs listed have no proven usefulness in ADHD.

Psychopharmacology

K&S Ch. 36

Question 140. C. Of the agents listed in this question, only sertraline (Zoloft) is FDA-approved in the pediatric population. It is only approved for obsessive–compulsive disorder (OCD) in patients aged 6 to 17 years. It is not indicated for depression or any of the other anxiety disorders in children. Fluoxetine (Prozac) is the only SSRI that is FDA-approved for the treatment of major depressive disorder in patients age 8 years and older. Fluvoxamine (Luvox) is also FDA-approved for OCD in patients 8 years of age and older. Paroxetine and venlafaxine are FDA-approved for depression and anxiety only in the adult population 18 years of age and older. Citalopram is FDA-approved for major depressive disorder only in adults over 18 years of age. Mirtazapine is only indicated for major depressive disorder and only in the adult population over 18 years of age.

Psychopharmacology

K&S Ch. 36

Question 141. A. This question contains a supportive statement that acknowledges how difficult things are for the patient currently, but gives her hope for the future. It is characteristic of supportive psychotherapy. In psychodynamic therapy and psychoanalysis, the therapist would wish to remain more neutral and not make statements that expressed opinion or boosted the patient’s mood. The question does not involve playing with children, so play therapy is incorrect. Cognitive behavioral therapy would involve looking at the patient’s assumptions and cognitions and seeing how they affect the patient’s mood. That is not happening in this question stem.

Supportive psychotherapy seeks to stabilize the self and the patient’s ability to cope. Defenses are strengthened. Symptom relief is sought. Neutrality is suspended. Direction by the therapist is encouraged. Free association is not part of this technique. Supportive psychotherapy can be used with those who have severe character pathology, psychosis, or are in the midst of an acute crisis or a physical illness.

Psychotherapy

K&S Ch. 35

Question 142. D. Risperidone decanoate injection (Risperdal Consta) is one of the atypical antipsychotics currently available in long-acting injectable form. The drug is FDA-approved for schizophrenia, particularly in noncompliant and refractory cases. The formulation comes in three dosing strengths: 25 mg, 37.5 mg, and 50 mg. The recommended dosing interval is 2 weeks. The product comes prepared in a dose-pack with sterile water in a vial and a syringe for reconstituting the drug. The powdered, unreconstituted drug must be refrigerated prior to adding water. Sterile water is necessary for reconstitution. Once reconstituted, the drug must be administered within 6 hours. The drug must only be injected to the gluteus muscle. The deltoid is not a recommended injection site because it is too small a muscle to accommodate proper timed dissipation of the agent. It is advisable to keep patients on their oral medication for at least three to four weeks while initiating risperidone Consta. This prevents possible decompensation from too rapid cross titration of antipsychotic agents.

Psychopharmacology

K&S Ch. 13; also see www.risperdalconsta.com

Question 143. C. Naloxone is an opioid antagonist. An opioid overdose is a medical emergency. Respiratory depression ensues leading to coma and shock. Naloxone is given IV and can be repeated 4–5 times in the first 30 minutes. Care must be used as the half-life is short and the patient can relapse back into coma, plus severe withdrawal can ensue from the use of an opioid antagonist.

Naltrexone is a longer acting antagonist with a half-life of 72 hours. It is a preventative measure for those with opioid addiction. It is used for blocking the euphoric effects of opioid use and thus decreases craving. Because of its long half life it is not the best choice for an emergency.

Buprenorphine is a mixed opioid agonist–antagonist. It is used in place of morphine to keep people off of heroin, and is not used in emergencies.

Benztropine is also known as Cogentin, an anticholinergic medication used to mitigate the side effects of antipsychotic drugs. It has nothing to do with opium overdose.

Bromocriptine is a mixed dopamine agonist–antagonist, which is approved in the US for treatment of Parkinson’s disease.

Psychopharmacology

K&S Ch. 12

Question 144. B. Elisabeth Kübler-Ross developed a comprehensive paradigm to classify the stages of a person’s reactions to impending death. Stage one is that of shock and denial. Upon learning the news that they are dying, people are initially in a state of shock and may deny that the diagnosis is correct. Stage two is that of anger. During this stage patients get frustrated, angry and irritable about their condition. They often ask: “Why me?” They typically undergo a lot of self-blame about their illness. Stage three is that of bargaining. Patients may try to negotiate or bargain with doctors, friends, family and even God to alleviate their illness in exchange for good deeds or fulfillment of certain pledges. Stage four is that of depression. During this stage patients demonstrate frank signs and symptoms of depression, including hopelessness, suicidal ideation, social withdrawal, and sleep problems. If the symptoms are severe enough to qualify as a major depressive disorder, the patient should be treated with an antidepressant. Stage five is that of acceptance. Patients acknowledge and come to terms with the inevitability of their death during this stage. Patients can begin to talk about facing the unknown without fear and with resolution.

Psychological Theory and Psychometric Testing

K&S Ch. 2

Question 145. D. Asking patients about suicide will have no effect on whether they are actually suicidal. Talking about suicide will not make patients suicidal. Most patients, when asked, are relieved to have permission to speak about something they have already thought about but were uncomfortable talking about with friends or family. The question is not frightening to them, nor will the mention of the word suicide help them plan their death.

Diagnostic and Treatment Procedures in Psychiatry

K&S Ch. 34

Question 146. D. Patients can display their grief over the death of loved ones in different ways and with different intensity. It is generally believed that a period of grief or mourning typically lasts about 6 months to 1 year. Some symptoms and signs of mourning may persist for a longer period, even up to 2 years or more. In most cases, the acute symptoms of grief improve over a period of about one to two months, after which time the individual returns to a more normal level of functioning.

Depressive Disorders

K&S Ch. 2

Question 147. D. The suicide rate for adolescents has quadrupled since 1950. Suicide accounts for 12% of deaths in the adolescent age group. The suicide rate has gone up more in this group than in any other group over the same time period.

Depressive Disorders

K&S Ch. 49

Question 148. D. Ramelteon (Rozerem) is a melatonin agonist and has a short half-life ranging from one to 2.5 hours. Ramelteon has an active metabolite, M-II, that has a half-life of about 5 hours. Zolpidem (Ambien) has a half-life of about 2.5 hours, but the duration of action can range from one to 4.5 hours. Zolpidem has no active metabolite. Zaleplon (Sonata) is a benzodiazepine receptor agonist that has the shortest half-life of all these agents at about one hour. It is therefore very useful for the treatment of middle insomnia. Eszopiclone (Lunesta) has the longest half-life of all these sleeping agents at about 6 hours. It is therefore, at least theoretically, the one most likely to cause next-day drowsiness. Triazolam (Halcion) is a benzodiazepine sedative–hypnotic agent with high potency and a short half-life ranging in duration from 2–4 hours.

Psychopharmacology

K&S Ch. 36

Question 149. A. The therapeutic range for valproic acid is 50–150 ng/mL. At doses as high as 125 ng/mL, side effects including thrombocytopenia may occur. Liver function tests should be obtained at the start of treatment and every 6–12 months thereafter, and valproic acid levels should be checked periodically.

Laboratory Tests in Psychiatry

K&S Ch. 7

Question 150. C. This is a question based on an understanding of the importance of culture. Different cultures view health and sickness differently. They view the medical system differently, and these differences must be taken into account when dealing with a multicultural population. In Japanese culture it is customary to minimize distress in front of an authority figure. This is the explanation for the patient’s behavior in this question. If the doctor involved was unaware of cultural issues, the patient’s behavior could be misinterpreted as psychosis or malingering, or the family could be seen as manipulative.

Cultural Issues in Psychiatry

K&S Ch. 4



If you find an error or have any questions, please email us at admin@doctorlib.org. Thank you!